CBSE Previous Year Solved Papers Class 12 Maths Outside Delhi 2016

CBSE Previous Year Solved  Papers Class 12 Maths Outside Delhi 2016

Time allowed: 3 hours                                                                                          Maximum Marks : 100
General Instructions:

  1. All questions are compulsory.
  2. Please check that this question paper contains 26 questions.
  3. Questions 1-6 in Section A are very short-answer type questions carrying 1 mark each.
  4. Questions 7-19 in Section B are long-answer I type questions carrying 4 marks each.
  5. Questions 20-26 in Section C are long-answer II type questions carrying 6 marks each.
  6. Please write down the serial number of the question before attempting it.

SET I

SECTION – A

Question numbers 1 to 6 carry 1 mark each.
Question.1.
cbse-previous-year-solved-papers-class-12-maths-outside-delhi-2016-1
Solution.
cbse-previous-year-solved-papers-class-12-maths-outside-delhi-2016-2

Question.2. Use elementary column operation C—>C2+2C1 in the following matrix equation:

cbse-previous-year-solved-papers-class-12-maths-outside-delhi-2016-3
Solution.
cbse-previous-year-solved-papers-class-12-maths-outside-delhi-2016-4

Question.3. Write the number of all possible matrices of order 2 x 2 with each entry 1,2 or 3.
Solution. Total number of all possible matrices of order 2 x 2 with each entry 1,2 or 3 are 3i.e., 81.

Question.4.
cbse-previous-year-solved-papers-class-12-maths-outside-delhi-2016-5
Solution.
cbse-previous-year-solved-papers-class-12-maths-outside-delhi-2016-6

Question.5. Write the number of vectors of unit length perpendicular to both the vectors
cbse-previous-year-solved-papers-class-12-maths-outside-delhi-2016-7
Solution.
cbse-previous-year-solved-papers-class-12-maths-outside-delhi-2016-8

Question.6. Find the vector equation of the plane with intercepts 3,-4 and 2 on x, y and z-axis respectively.
Solution.
cbse-previous-year-solved-papers-class-12-maths-outside-delhi-2016-9

SECTION – B

Question numbers 7 to 19 carry 4 marks each.
Question.7. Find the coordinates of the point where the line through the points A(3,4,1) and B(5,1,6) crosses the XZ plane. Also find the angle which this line makes with the XZ plane.
Solution.
cbse-previous-year-solved-papers-class-12-maths-outside-delhi-2016-10
cbse-previous-year-solved-papers-class-12-maths-outside-delhi-2016-11

Question.8.
cbse-previous-year-solved-papers-class-12-maths-outside-delhi-2016-12
Solution.
cbse-previous-year-solved-papers-class-12-maths-outside-delhi-2016-13
cbse-previous-year-solved-papers-class-12-maths-outside-delhi-2016-14
cbse-previous-year-solved-papers-class-12-maths-outside-delhi-2016-15

Question.9. In a game, a man wins Rs 5 for getting a number greater than 4 and loses Rs 1 otherwise, When a fair die is thrown. The man decided to throw a die thrice but to quit as and when he gets a number greater than 4. Find the expected value of the amount he wins/loses.
Solution. Let n denote the number of throws required to get a number greater than 4 and X denote the amount won/lost.
The man may get a number greater than 4 in the very first throw of the die or in second throw or in the third throw. Thus, we have the following probability distribution for X. ‘
cbse-previous-year-solved-papers-class-12-maths-outside-delhi-2016-16
OR
A bag contains 4 balls. Two balls are drawn at random (without replacement) and are found to be white. What is the probability that all balls in the bag are white ?
Solution. We know that the number of white balls can’t be less than 2.
cbse-previous-year-solved-papers-class-12-maths-outside-delhi-2016-17
cbse-previous-year-solved-papers-class-12-maths-outside-delhi-2016-18

Question.10. Differentiate xsin x + (sin x)cos x with respect to x.
Solution.
cbse-previous-year-solved-papers-class-12-maths-outside-delhi-2016-19
OR
cbse-previous-year-solved-papers-class-12-maths-outside-delhi-2016-20
Solution.
cbse-previous-year-solved-papers-class-12-maths-outside-delhi-2016-21

Question.11.
cbse-previous-year-solved-papers-class-12-maths-outside-delhi-2016-22
Solution.
cbse-previous-year-solved-papers-class-12-maths-outside-delhi-2016-23

Question.12. The equation of tangent at (2,3) on the curve y2 = ax3 + b is y = 4x-5. Find the value of a and b.
Solution.
cbse-previous-year-solved-papers-class-12-maths-outside-delhi-2016-24

Question.13.
cbse-previous-year-solved-papers-class-12-maths-outside-delhi-2016-25
Solution.
cbse-previous-year-solved-papers-class-12-maths-outside-delhi-2016-26
cbse-previous-year-solved-papers-class-12-maths-outside-delhi-2016-27

Question.14.
cbse-previous-year-solved-papers-class-12-maths-outside-delhi-2016-28
Solution.
cbse-previous-year-solved-papers-class-12-maths-outside-delhi-2016-29
cbse-previous-year-solved-papers-class-12-maths-outside-delhi-2016-30
cbse-previous-year-solved-papers-class-12-maths-outside-delhi-2016-31
OR
cbse-previous-year-solved-papers-class-12-maths-outside-delhi-2016-32
cbse-previous-year-solved-papers-class-12-maths-outside-delhi-2016-33

Question.15.
cbse-previous-year-solved-papers-class-12-maths-outside-delhi-2016-34
Solution.
cbse-previous-year-solved-papers-class-12-maths-outside-delhi-2016-35
cbse-previous-year-solved-papers-class-12-maths-outside-delhi-2016-36

Question.16. Solve the differential equation:
cbse-previous-year-solved-papers-class-12-maths-outside-delhi-2016-37
Solution.
cbse-previous-year-solved-papers-class-12-maths-outside-delhi-2016-38
cbse-previous-year-solved-papers-class-12-maths-outside-delhi-2016-39

Question.17. Form the differential equation of the family of circles in the second quadrant and touching the coordinate axes.
Solution.
cbse-previous-year-solved-papers-class-12-maths-outside-delhi-2016-40
cbse-previous-year-solved-papers-class-12-maths-outside-delhi-2016-41
This is the required differential equation representing the given family of circles.

Question.18. Solve the equation for x: sin-1 x + sin-1 (1 – x) = cos-1 x
Solution.
cbse-previous-year-solved-papers-class-12-maths-outside-delhi-2016-42
OR
cbse-previous-year-solved-papers-class-12-maths-outside-delhi-2016-43
Solution.
cbse-previous-year-solved-papers-class-12-maths-outside-delhi-2016-44
cbse-previous-year-solved-papers-class-12-maths-outside-delhi-2016-45

Question.19. A trust invested some money in two type of bonds. The first bond pays 10% interest and second bond pays 12% interest. The trust received Rs 2,800 as interest. However, if trust had interchanged money in bonds, they would have got Rs 100 less as interest. Using matrix method, find the amount invested by the trust. Interest received on this amount will be given to Help age India as donation. Which value is reflected in this question ?
Solution.
cbse-previous-year-solved-papers-class-12-maths-outside-delhi-2016-46
cbse-previous-year-solved-papers-class-12-maths-outside-delhi-2016-47

SECTION – C

Question numbers 20 to 26 carry 6 marks each.
Question.20. There are two types of fertilisers ‘A’ and ‘W. ‘A’ consists of 12% nitrogen and 5% phsophoric acid whereas ‘B’ consists of 4% nitrogen and 5% phsophoric acid. After testing the soil conditions, farmer finds that he needs at least 12 kg of nitrogen and 12 kg of phsophoric acid for his crops. If ‘A’ costs Rs 10 per kg and ‘B’ cost Rs 8 per kg, then graphically determine how much of each type of fertiliser should be used so that nutrient requirements are met at a minimum cost.
Solution.
cbse-previous-year-solved-papers-class-12-maths-outside-delhi-2016-48
cbse-previous-year-solved-papers-class-12-maths-outside-delhi-2016-49

Question.21. Five bad oranges are accidently mixed with 20 good ones. If four oranges are drawn one by one successively with replacement, then find the probability distribution of number of bad oranges drawn. Hence find the mean and variance of the distribution.
Solution. Let X denote the number of bad oranges in a draw of 4 oranges from a group of 20 good oranges and 5 bad oranges. Since there are 5 bad oranges in the group, therefore X can take values, 0,1,2,3,4.
Now, P(X = 0) = Probability of getting no bad orange. P(X = 0) = Probability of getting 4 good oranges
cbse-previous-year-solved-papers-class-12-maths-outside-delhi-2016-50
cbse-previous-year-solved-papers-class-12-maths-outside-delhi-2016-51

Question.22. Find the position vector of the loot of perpendicular and the perpendicular distance from the point P with position
cbse-previous-year-solved-papers-class-12-maths-outside-delhi-2016-52
Also find image of P in the plane.
Solution.
cbse-previous-year-solved-papers-class-12-maths-outside-delhi-2016-53
cbse-previous-year-solved-papers-class-12-maths-outside-delhi-2016-54
cbse-previous-year-solved-papers-class-12-maths-outside-delhi-2016-55

Question.23. Show that the binary operation *on A = R-{-l) defined as a*b = a + b + ab for all a, b e A is commutative and associative on A. Also find the identity element of * in A and prove that every element of A is invertible.
Solution.
cbse-previous-year-solved-papers-class-12-maths-outside-delhi-2016-56
cbse-previous-year-solved-papers-class-12-maths-outside-delhi-2016-57
cbse-previous-year-solved-papers-class-12-maths-outside-delhi-2016-58

Question.24. Prove that the least perimeter of an isosceles triangle in which a circle of radius r can be inscribed is 6√3r.
Solution.
cbse-previous-year-solved-papers-class-12-maths-outside-delhi-2016-59
cbse-previous-year-solved-papers-class-12-maths-outside-delhi-2016-60
cbse-previous-year-solved-papers-class-12-maths-outside-delhi-2016-61
OR
If the sum of lengths of hypotenuse and a side of a right angled triangle is given, show that area of triangle is maximum, when the angle between them is π/3.
Solution.
cbse-previous-year-solved-papers-class-12-maths-outside-delhi-2016-62
cbse-previous-year-solved-papers-class-12-maths-outside-delhi-2016-63
cbse-previous-year-solved-papers-class-12-maths-outside-delhi-2016-64

Question.25. Prove that the curves y2 = 4x and x2 = 4y divide the area of square bounded by x = 0, x = 4, y = 4 and y = 0 into three equal parts.
Solution.
cbse-previous-year-solved-papers-class-12-maths-outside-delhi-2016-65
cbse-previous-year-solved-papers-class-12-maths-outside-delhi-2016-66

Question.26. Using properties of determinants, show that ∆ABC is isosceles if:
cbse-previous-year-solved-papers-class-12-maths-outside-delhi-2016-67
Solution.
cbse-previous-year-solved-papers-class-12-maths-outside-delhi-2016-68
cbse-previous-year-solved-papers-class-12-maths-outside-delhi-2016-69
OR
A shopkeeper has 3 varieties of pens ‘A’, ‘B’ and ‘C’. Meenu purchased 1 pen of each variety for a total of Rs 21. Jeevan purchased 4 pens of ‘A’ variety, 3 pens of ‘B’ variety and 2 pens of ‘C’ variety for Rs 60. While Shikha purchased 6 pens of’A’ variety, 2 pens of ‘W variety and 3 pens of ‘C’ variety for Rs 70. Using matrix method, find cost of each variety of pen.
Solution.
cbse-previous-year-solved-papers-class-12-maths-outside-delhi-2016-70
cbse-previous-year-solved-papers-class-12-maths-outside-delhi-2016-71
cbse-previous-year-solved-papers-class-12-maths-outside-delhi-2016-72

CBSE Solved Papers For Class 12 Computer Science (C++) Paper 2

CBSE Solved Papers For Class 12 Computer Science (C++) Paper 2

SECTION A

Question 1.
(a) Define Macro with suitable example.
(b) Which C++ header file (s) will be included to run /execute the following C++ code?

void main( )
 {
 int Last =26.5698742658;
 cout< }

(c) Rewrite the following program after removing any syntactical errors. Underline each correction made.

#include
 void main( )
 int A[10];
 A=[3, 2, 5, 4, 7, 9, 10]; int S = 0, p;
 for ( p = 0; p<=6; p++)
 { if(A[p]%2=0)
 int S = S+A[p]; }
 cout<<S;
 }

(d) Find the output of the following C+ + program:

#include
 void repch(char s[])
 {
 for (int i=0;s [i] !='\0';i++)
 {
 if(((i%2) !=0) &&(s[i]!=s[i+1] ) )
 {
 s[i]='@';
 }
 else if (s [i]==s[i+1])
 {
 s[i+l]='!';
 i++;
 }
 }
 }
 void main()
 {
 char str[]="SUCCESS";
 cout<<"Original String"<<str
 repch(str);
 cout<<"Changed String"<<str;
 }

(e) Find the output of the following :

#include
 void switchover(int A[ ],int N, int split)
 {
 for(int K = 0; K<N; K++)
 if(K<split)
 A[K] += K;
 else
 A[K]*= K;
 }
 void display(int A[ ] ,int N)
 {
 for (int K = 0; K<N; K++)
 (K%2== 0) ?cout<<A[K]<<"%" : cout< }
 void main( )
 { int H[ ] = {30, 40, 50, 20, 10, 5};
 switchover(H, 6, 3);
 display(H, 6);
 }

(f) Observe the following C++ code and find out, which out of the given options (i) to (iv) are the expected correct outputAlso assign the maximum and minimum value that can be assigned to the variable ‘Go’.

void main()
 { int X [4] ={100,75,10,125};
 int Go = random(2)+2;
 for (inti = Go; i< 4; i++)
 cout<<X[i ]<<"$$";
 }
 i. 100$$75 ii. 75$$10$$125$$ iii. 75$$10$$ iv.l0$$125$

Answer:
(a) Macros are preprocessor directive created using # define that serve as symbolic constants. They are created to simplify and reduce the amount of repetitive coding,

For instance,
 ##define max (a, b) (a>b? a: b)

Defines the macro max, taking two arguments a and b. This macro may be called like any function. Therefore, after preprocessing

A = max (x, y) ;
 Becomes A = (x>y?x :y) ;

(b) iostream.h
iomanip.h

(c) #include
 void main( )
 { int A[10] = {3, 2, 5, 4, 7, 9, 10};
 int S = 0,p;
 for(p = 0; p<=6; p++)
 { if(A[p]%2==0) (
 S = S+A [p] ; }
 Cout<<S;
 }

(d) Original String SUCCESS
Changed String S@C!ES!

(e) 30%41
52% 60
40% 25

(f) iv is the correct option.
Minimum value of Go = 2
Maximum value of Go = 3

Question 2.
(a) Differentiate between data abstraction and data hiding.
(b) Answer the questions (i) and (ii) after going through the following class :

class Exam
 {
 int Rollno;
 char Cname[25];
 float Marks ;
 public :
 Exam( ) //Function 1
 {
 Rollno = 0 ;
 Cname="";
 Marks=0.0;
 }
 Exam(int Rno, char candname) //Function 2
 }
 Rollno = Rno ;
 strcpy(Cname,candname);
 }
 -Exam() //Function 3
 cout << "Result will be intimated shortly" < }
 void() display( ) //Function 4
 cout << "Roll no :"<<Rollno;
 cout<<"Name <<Cname;
 cout <<" Marks:"<<Marks; } } ;

(i) Which OOP concept does Function 1 and Function 2 implement.Explain?
(ii) What is Function 3 called? When will it be invoked?
(c) Define a class Candidate in C++ with the following specification : Private Members: A data members Rno(Registration Number) type long A data member Cname of type string A data members Agg_marks (Aggregate Marks) of type float A data members Grade of type char A member function setGrade () to find the grade as per the aggregate marks obtained by the student. Equivalent aggregate marks range and the respective grade as shown below. Aggregate Marks Grade >=80 A
Less than 80 and > =65 B
Less than 65 and > =50 C
Less than 50 D
Public members:
A constructor to assign default values to data members :
Rno=0,Cname=”N.A”,Agg_marks=0.0
A function Getdata () to allow users to enter values for Rno. Cname, Agg marks and call function
setGrade () to find the grade.
A function dispResult() to allow user to view the content of all the data members.
(d) Give the following class definition answer the question that is follow:

class University
 {
 char name [20];
 protected :
 char vc[20];
 public :
 void estd();
 void inputdata();
 void outputdata() ;
 }
 class College : protected University
 { int regno;
 protected
 char principal ()
 public :
 int no_of_students;
 void readdata();
 void dispdata ( );
 } ;
 class Department : public College
 char name[20];
 char HOD[20];
 public :
 void fetchdata(int);
 void displaydata( );
 }

(i) Name the base class and derived class of college.
(ii) Name the data member(s) that can be accessed from function displaydata().
(iii) What type of inheritance is depicted in the above class definition?
(iv) What will be the size of an object (in bytes) of class Department?

Answer:
(a) Data hiding can be defined as the mechanism of hiding the data of a class from the outside world. This is done to protect the data from any accidental or intentional access.
Implementation-Data hiding is achieved by making the members of the class private.
Data abstraction refers to, providing only essential information to the outside world and hiding their background details.
Implementation-Members defined with a public label are accessible to all parts of the program. The dataabstraction view of a type is defined by its public members.
(b) (i) Constructor Overloading/Polymorphism, as multiple definitions for Constructors are given in the same scope. Functiort 1 is a Default constructor and function 2 is a Parameterized constructor.
(ii) Function 3 is a Destructor which is invoked when the object of class goes out of scope.

(c) class Candidate
 { long Rno;
 char Cname[20];
 float Agg_marks;
 char Grade;
 void setGrade()
 { if (Agg_marks>= 80)
 Grade = 'A';
 else if (Agg_marks>=65)
 Grade = 'B';
 else if (Agg_marks>=50)
 Grade ='C';
 else
 Grade='D';
 }
 public:
 Candidate()
 {
 Rno=0;
 Strcpy(Cname,"N.A.");
 Agg_marks=0.0;
 }
 void Getdata ()
 { cout<<"Registration No"; cin>>Rno;
 cout<<"Name";
 gets(name);
 cout<>Agg_marks;
 setGrade();
 }
 void dispResult()
 {
 cout<<"Registration No"<<Rno;
 cout<<"Name"puts(cname);
 cout< }

(d) (i) Base class: University
Derived class: Department
(ii) name[20], HOD[20], no_of_students.
[Hint: By default access type is private]
(iii) Multilevel Inheritance
[1 mark for the correct answer]
(iv) 62 bytes
[Note: 65 will be also considered correct because of NULL character i.e., ’10’ at last of each array. So ultimately size of array is 21],
85 bytes

Question 3.
(a) An integer array A [40] [30] is stored along the row in the memory. If the element A[20][25] is stored at 50000, find out the location of A[7][10].]
(b) Write the definition of function insert for the linked implemented queue containing passenger in-formation as follows: [4]

struct NODE
 { int Ticketno;
 char PName[20];
 NODE * NEXT; };
 class Queueofbus
 { NODE *Rear, *Front;
 public:
 Queueofbus()
 { Rear = NULL;
 Front = NULL; };
 void Insert ();
 void Delete();
 -Queueofbus()
 { cout<<"Object destroyed"; }
 } ;

(c) Write a function to sort any array of n elements using insertion sort. Array should be passed as argument to the function.
(d) Write a function NewMAT(int A[][],int r,int c ) in C++, which accepts a 2d array of integer and its size as parameters divide all those array elements by 6 which are in the range 60 to 600(both values inclusive) in the 2d Array .
(e) Evaluate the following postfix expression using stack and show the contents after execution of each Operations:470, 5, 4, ^ , 25, /, 6, *

Answer:
(a) A[i] [j] = B+W x [No.of columns x(I-Lr) + (J-L,) ]
A [20] [25] = B+ 2x[30x(20-0) + (25-0) ]
50000 = B+2x[30x(20-0)+(25-0)]
B = 48750
A[7] [10] = 48750+ 2x[30x(7-0) + (10-0)]
= 49190

(b) void Queueofbus::Insert()
 { NODE *p = new NODE;
 cout<<"Enter Ticket no" cin>>p->ticketno;
 cout<<"Enter Name"; cin>>p->Pname;
 p->NEXT = NULL;
 if (rear == NULL)
 { Rear = p;
 Front = Rear;
 }
 else
 { Rear -> NEXT = p;
 Rear = Rear -> NEXT;
 }
 }
(c) void insertsort( int a[],int n)
 {
 int p,ptr;
 //Assuming a[0]=int_min i.e. smallest integer
 for (p=1 ,p<=n;p+ + )
 {
 temp=a[p];
 ptr=p-1;
 while(temp<a[ptr])
 {
 a [ptr+1]=a[ptr]; // Move Element Forward
 ptr- -;
 }
 a[ptr+1]=temp; // Insert Element in Proper Place
 }
(d) void NewMAT(int A[][],int r,int c)
 {
 for (int i = 0;i<r;i++)
 for(j =0;j <c;j ++) if ( (A [i] [ j ] >=60 )&&(A[i] [ j ] <=600) )
 A [i] [j]/=6 ;
 or
 A [i] [j] = A [i] [ j ] / 6 ;
 }

(e)

S.No. Symbol Operation  Stack  Result
1 470 push(470) 470
2 5 push(5) 470, 5
3 4 push(4) 470, 5, 4
4 A pop (4) 470, 5
pop(5) 470
perform(5ˆ4)
push(625) 470, 625
5 25 push(25) 470, 625, 25
6 / pop(25) 470, 625
pop(625) 470
perform(625/25) 470
push(25) 470, 25
7 6 push(6) 470, 25, 6
8 * pop(6) 470, 25
pop(25) 470
perform(25*6) 470

Question 4.
(a) Consider a file F containing objects E of class Emp.
(i) Write statement to position the file pointer to the end of the file
(ii) Write statement to return the number of bytes from the beginning of the file to the current posi¬tion of the file pointer.
(b) Write a function RevText() to read a text file ” Input.txt ” and Print only word starting with T in reverse order.
Example: If value in text file is: INDIA IS MY COUNTRY Output will be: AIDNI SI MY COUNTRY
(c)Write a function in C+ + to search and display details, whose destination is “Chandigarh”from binary file “Flight.Dat”. Assuming the binary file is containing the objects of the following class:

class FLIGHT
 { int Fno; // Flight Number
 char From[20j; // Flight Starting Point
 char To[20j; // Flight Destination
 public:
 char * GetFrom (); { return from; }
 char * GetTo(); { return To; }
 void input() { cin> >Fno> >; gets(From); gets(To); }
 void show() { cout< <Fno< < "From:"; puts (From); cout < < "\n To:"; puts (To); }
 };

Answer:
(a) (i) F.seekg(0,ios::end);
(ii) F.tellg();

(b) void RevText()
 { ifstream in ("Input.txt");
 char word[25];
 while(in)
 { in>>word;
 if (word[0]=='I')
 cout<<strrev(word)<<..;
 else
 cout<<word<<....;
 }
(c) void Dispdetails()
 { ifstream fin ( "Flight. Dat" ) ;
 Flight F;
 while (fin)
 { fin. read( (char*) &F, sizeof (F) )
 if (strcmp(F.GetTo(),"Chandigarh"))
 F.show();
 }
 }

SECTION B

Question 5.
(a) Differentiate between cardinality and degree of a table with the help of an example.
(b) Consider the following tables FACULTY and COURSES. Write SQL commands for the statements (i) to (v) and give outputs for SQL query (vi)
FACULTY

F_ID Fname Lname Hire_date Salary
102 Amit Mishra 12-10-1998 12000
103 Nitin Vyas 24-12-1994 8000
104 Rakshit Soni 18-5-2001 14000
105 Rashmi Malhotra 11-9-2004 11000
106 Sulekha Srivastava 5-6-2006 10000

COURSES

C_ID F_ID Cname Fees
C21 102 Grid Computing 40000
C22 106 System Design 16000
C23 104 Computer Security 8000
C24 106 Human Biology 15000
C25 102 Computer Network 20000
C26 105 Visual Basic 6000

(i) To display details of those Faculties whose salary is greater than 12000.
(ii) To display the details of courses whose fees is in the range of 15000 to 50000 (both values included).
(iii) To increase the fees of all courses by 500 of “System Design” Course.
(iv) To display details of those courses which are taught by ‘Sulekha’ in descending order of courses.
(v) Select COUNT (DISTINCT F_ID) from COURSES;
(vi) Select MIN(Salary) from FACULTY,COURSES where COURSES.F ID = FACULTY.F_ID;

Answer:
(a) Cardinality is defined as the number of rows in a table. Degree is the number of columns in a table.
Eg:Consider the following tables:
Table : Account

Acno Cname
Ac100 Sheela
Ac101 Darsh
Ac102 Kathy

Cardinality of Account table is : 4
Degree of Account table is :2

(b) (i) Select * from faculty
where salary > 12000
(ii) Select * from Courses
where fees between 15000 and 50000
(iii) Update courses set fees = fees + 500
where Cname = “System Design”
(iv) Select * from faculty fac,courses cour
where fac.f_id = cour.f_id
and fac.fname = ‘Sulekha’
order by cname desc
(v) 4
(vi) 12000

Question 6.
(a) State and Verify Absorption law algebraically.
(b) Draw a logic circuit for the following Boolean expression: A.B+C.D’.
(c) Write the SOP form of a Boolean function F, which is represented in a truth table as follows:

A B C D
0 0 0 0
0 0 1 1
0 1 0 1
0 1 1 0
1 0 0 1
1 0 1 1
1 1 0 0
1 1 1 0

(d) Obtain a simplified form for a Boolean expression:
F (U, V, W, Z) = 7i (0,1, 3,5,6, 7,15)

Answer:
(a) Absorption law states that:
A + AB = A and A . (A + B) = A
Algebraic method:
Taking LHS
A + AB = (A.1) + (A.B) by Identity
= A. (1+B) by Distribution
= A.1 by Null Element
= A
(b)
cbse-solved-papers-for-class-12-computer-science-c-paper-2-2
(c) ABC’+AB’C+A’BC+A’BC [1 mark for all the terms]
(d) (u+v+w).(u+z’).(v’+w’).(u’+w’+z)
cbse-solved-papers-for-class-12-computer-science-c-paper-2-3

Question 7.
(a) Write any 1 advantage and 1 disadvantage of Bus topology.
(b) SunRise Pvt. Ltd. is setting up the network in the Ahmadabad. There are four departments named as MrktDept, FunDept, LegalDept, SalesDept.
cbse-solved-papers-for-class-12-computer-science-c-paper-2-1
Distance between various buildings is given as follows:

MrktDept to FunDept 80 m
MrktDept to LegalDept 180m
MrktDept to SalesDept 100 m
LegalDept to SalesDept 150 m
LegalDept to FunDept 100 m
FunDept to SalesDept 50 m

Number of Computers in the buildings:

MrktDept 20
LegalDept 10
FunDept 08
SalesDept 42

(i) Suggest a cable layout of connections between the Departments and specify topology.
(ii) Suggest the most suitable building to place the server with a suitable reason.
(iii) Suggest the placement of i) modem ii) Hub /Switch in the network.
(iv) The organization is planning to link its sale counter situated in various part of the same city/which type of network out of LAN, WAN, MAN will be formed? Justify.
(c) Name the protocol
(i) Used to transfer voice using packet switched network.
(i) Used for chatting between 2 groups or between 2 individuals.
(d) What is an IP Address?
(e) What is HTTP?
(f) Explain the importance of Cookies.
(g) How is 4G different from 3G?

Answer:
(a) Advantage: Since there is a single common data path connecting all the nodes, the bus topology uses a very short cable length which considerably reduces the installation cost.
Disadvantage: Fault detection and isolation is difficult. This is because control of the network is not centralized in any particular node. If a node is faulty on the bus, detection of fault may have to be performed at many points on the network. The faulty node has then to be rectified at that connection point.
(b) (i)
cbse-solved-papers-for-class-12-computer-science-c-paper-2-4
Star Topology should be used.
(ii)As per 80 – 20 rule, Mrkt Dept beacause it has maximium no. of computers.
(iii) Each building should have hub/switch and Modem in case Internet connection is required.
(iv) MAN (Metropolitan Area Network). Because it is suitable to correct across a city.
(c) (i) VOIP (Voice Over Internet Protocol)
(ii) IRC(Internet Relay Chat)
(d) An IP address is a unique identifier for a node or host connection on an IP network. An IP address
is a 32 bit binary number usually represented as 4 decimal values, each representing 8 bits, in the range 0 to 255 (known as octets) separated by decimal points. This is known as “dotted decimal” notation. ’
Example:140.179.220.200
(e) HTTP is a protocol that is used for transferring hypertext(i.e. text,graphic,image,sound,video,etc,) between 2 computers and is particularly used on the World Wide Web (WWW). It is acronym for Hyper Text Transfer Protocol.
(f) When the user browses a website, the web server sends a text file to the web browser. This small text file is a cookie. They are usually used to track the pages that we visit so that information can be customised for us for that visit.
(g) 3G technology adds multimedia facilities such as video,audio and graphics applications whereas 4G will provide better than TV quality images and video-links. Transfer rate of 4G is very high as compared to 3G.

Class 8 Geography Chapter 5 Extra Questions and Answers Industries

Class 8 Geography Chapter 5 Extra Questions and Answers Industries

In this chapter, “Class 8 Geography Chapter 5 Extra Questions and Answers Industries,” we’ll traverse through the vast domains of factories and production units, discovering the various types of industries and their role in economic development. From understanding the concept of industrialization to exploring the challenges and opportunities in the industrial sector, this narrative unveils the significance of industries in shaping modern societies. Read this also Extra Questions for Class 8 Social Science .

Class 8 Geography Chapter 5 Extra Questions and Answers Industries

Industries Class 8 Extra Questions and Answer Geography Chapter 5 Very Short Answers Type

Question 1.
What is the basic function of secondary activities or manufacturing?
Answer:
The basic function of secondary activities or manufacturing is to change raw materials into products of more value to people.

Question 2.
Of what is the paper made up from?
Answer:
The paper is made up from pulp.

Question 3.
Name the classification of industries on the basis of raw material used by them.
Answer:
The industries classified on the basis of raw materials used by them are:

  1. Agro-based industries
  2. Mineral-based industries
  3. Marine based industries
  4. Forest-based industries

Question 4.
By whom are the private sector industries owned and operated?
Answer:
Private-sector industries are owned and operated by individuals or a group of individuals.

Question 5.
What does an industrial system consist of?
Answer:
An industrial system consists of input, processes and outputs.

Question 6.
How does an industrial region emerge?
Answer:
Industrial region emerges when a number of industries are located close to each other and share the benefits of their closeness.

Question 7.
What is meant by industrial disaster?
Answer:
In industries, accidents/disasters mainly occur due to technical failure or irresponsible handling of hazardous material. This is known as industrial disaster.

Question 8.
Define Sunrise Industries.
Answer:
Emerging industries are also known as ‘Sunrise Industries’. These include Information technology, Wellness, Hospitality and Knowledge.

Question 9.
Define the process ‘smelting’.
Answer:
It is the process in which metals are extracted from their ores by heating beyond the melting point.

Question 10.
What is the full form of TISCO?
Answer:
TISCO: Tata Iron and Steel Company Limited.

Question 11.
On what basis are industries classified?
Answer:
Industries are classified on the basis of raw materials, size and ownership.

Question 12.
Under which category is production of automobile and heavy machinery is placed?
Answer:
Under large scale industry.

Question 13.
What may be the inputs in case of textile industry?
Answer:
In case of textile industry, the inputs may be cotton, human labour, factory and transport cost.

Question 14.
What was the official death total in 1989 in Bhopal tragedy?
Answer:
Official death total was 3,598 in 1989.

Question 15.
What is MIC?
Answer:
MIC is Methyl Isocynate gas responsible for gas tragedy of Bhopal.

Question 16.
Which is the major IT hub of India?
Answer:
Bengaluru.

Question 17.
Where was TISCO started in?
Answer:
It was started in Sakchi.

Question 18.
What is the date of Bhopal gas tragedy?
Answer:
On 3rd December 1984.

Question 19.
Which industries use produce from sea and oceans as raw materials?
Answer:
Marine based industries.

Question 20.
In which part of the world is silicon valley located?
Answer:
It is located next to the rocky mountains of North America.

Industries Class 8 Extra Questions and Answer Geography Chapter 5 Short Answers Type

Question 1.
Why was Sakchi chosen to set up the steel plant? Give reasons.
Answer:
Sakchi was chosen to set up the steel plant for several reasons because:

  1. the place was only 32 km away from Kalimati station on the Bengal-Nagpur railway line.
  2. It was close to the iron ore, coal and manganese deposits as well as to Kolkata, which provided a large market.
  3. TISCO gets coal from Jharia coalfields, and iron ore, limestone, dolomite and manganese from Odisha and Chattisgarh.
  4. The Kharkai and Subarnarekha rivers ensured sufficient water supply. Government initiatives provided adequate capital for its later development.

Question 2.
What was the ideal location for iron and steel industry before 1800 A.D and after 1950 how did the concept for ideal location of iron and steel industries change?
Answer:
Before 1800 A.D. iron and steel industry was located where raw materials, power supply and running water were easily available. Later, the ideal location for the industry was near coal fields and close to canals and railways.

After 1950, iron and steel industry began to be located on large areas of flat land near sea ports. This is because by this time steelworks had become very large and iron ore had to be imported from overseas.

Question 3.
How is the steel used by other industries as raw materials?
Answer:
Steel is used by other industries as raw material in many ways.

  1. Steel is tough and it can be easily shaped, cut, or made into wire.
  2. Special alloys of steel can be made by adding small amount of other metals such as aluminium, nickel and copper.
  3. Alloys give steel unusual hardness, toughness or ability to resist rust.
  4. Steel is often called the backbone of modern industry.

Question 4.
Define the concept of industrial system briefly.
Answer:
An industrial system consists of inputs, processes and outputs. The inputs are the raw materials, labour and cost of land, transport, power and other infrastructure. The processes include a wide range of activities that convert the raw material into finished products. The outputs are the end products and the income earned from it.

In case of textile industry the inputs may be cotton, human labour, factory and transport cost. The processes include ginning, spinning, weaving, dyeing and printing. The output is the shirt you wear.

Question 5.
What does the concept of information technology industry deals in and which are the major hubs of the IT industry?
Answer:
The information technology industry deals in the storage, processing and distribution of information. Today, this industry has become global. This is due to a series of technological, political and socio-eco¬nomic events.

The main factors guiding the location of these industries are resource availability, cost and infrastructure. The major hubs of the IT industry are the Silicon Valley, California and Bengaluru, India.

Industries Class 8 Extra Questions and Answer Geography Chapter 5 Long Answers Type

Question 1.
How is the classification of industries done on the basis of raw materials size and ownership? Explain.
Answer:
Industries are classified on the basis of raw materials, size and ownership:
1. Raw materials: Industries may be agro-based, mineral-based, marine-based and forest-based depending on the type of raw materials used by them.

  • Agro-based industries: These industries use plant and animal-based products as their raw materials.
  • Mineral-based industries: These industries use mineral ores as their raw materials, and the products of these industries feed other industries.
  • Marine based industries: They use the products from the sea and oceans as raw materials.
  • Forest-based industries: It utilises forest produce as raw materials.

2. Size: It refers to the amount of capital invested, number of people employed and the volume of production.
Based on size, industries can be classified into small scale and large scale industries.

  • Small scale industry: Cottage or household industries are a type of small scale industry where the products are manufactured by hand, by the artisans. These industries use lesser amount of capital and technology as compared to large scale industry.
  • Large scale industry: These are industries that produce large volumes of products. Investment of capital is higher and the technology used is superior in large scale industries.

3. Ownership: Industries can be classified into private sector, state-owned or public sector, joint sector and cooperative sector.

  • Private-sector industries: These are owned and operated by individuals or a group of individuals.
  • Public sector industries: These are owned and operated by the government.
  • Joint sector industries: These are owned and operated by the state and individuals or a group of individuals.
  • Cooperative sector industries: The are owned and operated by the producers or suppliers of raw materials, workers or both.

Picture Based Questions Class 8 Geography Chapter 5 Industries

Look at the picture given below and answer the following questions:
Picture-1
Class 8 Geography Chapter 5 Extra Questions and Answers Industries img-1
(i) What does the above picture show?
(ii) Where was the iron and steel industry located before 1800 A.D.?
Answer:
(i) It shows the changing location of iron and steel industry.
(ii) Before 1800 A.D. iron and steel industry was located where raw materials, power supply and running water were easily available.

Picture-2
Class 8 Geography Chapter 5 Extra Questions and Answers Industries img-2
(i) Name the major railway station shown in the figure.
(ii) What does the picture show?
Answer:
(i) Tatanagar railway station.
(ii) It shows the location of iron and steel industry in Jamshedpur.

Map Skills Class 8 Geography Chapter 5 Industries

Question 1.
On an outline Map of World show the major iron ore producing areas.
(i) Brazil
(ii) South Africa
(iii) Australia
(iv) Germany
(v) India
Answer:
Class 8 Geography Chapter 5 Extra Questions and Answers Industries img-3

Question 2.
On an outline Map of World show the important iron and steel producing centres.
(i) Jharkhand
(ii) Odisha
(iii) Chhattisgarh
(iv) West Bengal
Answer:
Class 8 Geography Chapter 5 Extra Questions and Answers Industries img-4

CBSE Previous Year Solved Papers Class 12 Computer Science Outside Delhi 2013

CBSE Previous Year Solved  Papers  Class 12 Computer Science Outside Delhi 2013

Time allowed : 3 hours                                                                                           Maximum Marks: 70

General Instructions :

  1.  There are a total of 26 questions and five sections in the question paper, All questions are compulsory.
  2. Section A contains question number 1 to 5, Very Short Answer type questions of one mark each.
  3.  Section B contains question number 6 to 10, Short Answer type I questions of two marks each.
  4.  Section C contains question number 11 to 22, Short Answer type II questions of three marks each.
  5.  Section D contains question number 23, Value Based Question of four marks.
  6. Section E contains question number 24 to 26, Long Answer type questions of five marks each.
  7. There is no overall choice in the question paper, however, an internal choice is provided in one question of two marks, one question of three marks and all three questions of five marks. An examined is to attempt any one of the questions out of two given in the question paper with the same question number.

SET I

Question.1. (a) What is the benefit of using default parameter/argument in a function? Give a suitable example to illustrate it using C++ code
Answer : The benefits of using default parameter/argument in a function are as following:

  • They can be used to add new parameters to existing function.
  • They can be used to combine similar function into one.

cbse-previous-year-solved-papers-class-12-computer-science-outside-delhi-2013-1

(b) Observe the following C++ code and write the name(s) of the header file(s), which will be essentially required to run it in a C++ compiler:
cbse-previous-year-solved-papers-class-12-computer-science-outside-delhi-2013-2

(c) Observe the following C++ code carefully and rewrite the same after removing all the syntax error(s) present in the code. Ensure that you underline each correction in the code.
Important Note:
• All the desired header files are already included, which are required to run the code.
• Correction should not change the logic of the program.
cbse-previous-year-solved-papers-class-12-computer-science-outside-delhi-2013-3

(d) Observe the following C++ code carefully and obtain the output, which will appear on the screen after execution of it.
Important Note:
• All the desired header files are already included in the code, which are required to run the code.
cbse-previous-year-solved-papers-class-12-computer-science-outside-delhi-2013-4

(e) Observe the following C++ code carefully and obtain the output, which will appear on the screen after execution of it.
cbse-previous-year-solved-papers-class-12-computer-science-outside-delhi-2013-5
cbse-previous-year-solved-papers-class-12-computer-science-outside-delhi-2013-6
cbse-previous-year-solved-papers-class-12-computer-science-outside-delhi-2013-7

(f) Based on the following C++ code find out the expected correct output(s) from the option (i) to (iv). Also, find out the minimum and the maximum value that can be assigned to the variable Guess used in the code at the time when value of Turn is 3.
cbse-previous-year-solved-papers-class-12-computer-science-outside-delhi-2013-8
cbse-previous-year-solved-papers-class-12-computer-science-outside-delhi-2013-9

Question.2. (a) Write any two similarities between Constructor and
Destructor. Write the (unction headers for constructor and destructor of a class Flight.
Answer:
Similarities :
(i) Constructors and destructor’s do not have any return type, not even void, nor can they return values.
(ii) References and pointers cannot be used on constructors and destructor’s because their addresses cannot be taken.
cbse-previous-year-solved-papers-class-12-computer-science-outside-delhi-2013-10

(b) Answer the questions (i) and (ii) alter going through the following class:
cbse-previous-year-solved-papers-class-12-computer-science-outside-delhi-2013-11
(i) Out of the following, which of the option is correct for calling Function 2?
(ii) Option 1-Race T(30);
(iii) Option 2-Race U(R);
(iv) Name the feature of Object Oriented Programming, which illustrated by Function 1, Function 2 and Function 3 combined together.
Answer:
(i) Option-1 Race T (30)
(ii) Constructor overloading.

(c) Define a class Bus. in C++ with the following specifications:
Data Members
• Busno – to store Bus No
• From to store Place name of origin
• To — to store Place name of destination
• Type — to store Bus Type such as ‘O’ for ordinary
• Distance – to store the Distance in Kilometres
• Fare —to store the Bus Fare
Member Functions
• A constructor function to initialize Type as ‘O’ and Freight as 500 ’
• A function CalcFare() to calculate Fare as per the following criteria:
TYPE FARE
‘O’ 15*Distance
‘E’ 20*Distance
‘L’ 24*Distance
• A function Allocate)) to allow user to enter values for Busno, From, To, Type and Distance. Also, this function should call CalcFare() to calculate Fare.
• A function Show() to display the content of all the data members on screen.
cbse-previous-year-solved-papers-class-12-computer-science-outside-delhi-2013-12
cbse-previous-year-solved-papers-class-12-computer-science-outside-delhi-2013-13
cbse-previous-year-solved-papers-class-12-computer-science-outside-delhi-2013-14
cbse-previous-year-solved-papers-class-12-computer-science-outside-delhi-2013-15

(d) Consider the following C++ code and answer the questions from (i) to (iv):
cbse-previous-year-solved-papers-class-12-computer-science-outside-delhi-2013-16
cbse-previous-year-solved-papers-class-12-computer-science-outside-delhi-2013-17
(i) Which type of inheritance is shown in the above example?
(ii) Write the names of those data members, which can be directly accessed from the objects of class Result.
(iii) Write the names of those member functions which can be directly accessed from the objects of class Result.
(iv) Write names of those data members, which can be directly accessed from the Mentry () function of class Marks.
cbse-previous-year-solved-papers-class-12-computer-science-outside-delhi-2013-18

Question.3. (a) Write code for a function void Chang Over (inf P[ ],int N) in C++, which re-positions all the elements of the array by shifting each of them to the nest position and by shifting the last element to the first position.
cbse-previous-year-solved-papers-class-12-computer-science-outside-delhi-2013-19

(b) An array T[15][10] is stored along the row in the memory with each element requiring 8 bytes of storage. If the base address of array T is 14000, find out the location of T[ 10] [7].
cbse-previous-year-solved-papers-class-12-computer-science-outside-delhi-2013-20

(c) Write a user defined function DispTen(int A[ ] [4], int N, int M) In C++ to find and display all the numbers, which are divisible by 10. For example if the content of array is:
cbse-previous-year-solved-papers-class-12-computer-science-outside-delhi-2013-21
The output should be 20 10 30
cbse-previous-year-solved-papers-class-12-computer-science-outside-delhi-2013-22

(d) Evaluate the following post fix expression. Show the status of stack after execution of each operation: 5, 2, *, 50, 5, /, 5,-,+
Answer:
cbse-previous-year-solved-papers-class-12-computer-science-outside-delhi-2013-23

(e) Write a function Q DELETE() in C++ to perform delete operation on a Linked Queue, which contains Passenger no and Passenger name. Consider the following definition of node in the code.
cbse-previous-year-solved-papers-class-12-computer-science-outside-delhi-2013-24
cbse-previous-year-solved-papers-class-12-computer-science-outside-delhi-2013-25

Question.4. (a) Fill in the blanks marked as Statement 1 and Statement 2, in the program segment given below with appropriate functions for the required task.
cbse-previous-year-solved-papers-class-12-computer-science-outside-delhi-2013-26
cbse-previous-year-solved-papers-class-12-computer-science-outside-delhi-2013-27

(b) Write a function Count You Me() in C++ which reads the contents of a text file story.txt and counts the words You and Me (not case sensitive).
For example, if the file contains:
You are my best friend.
You and me make a good team.
The function should display the output as Count for You: 2 Count for Me: 1
cbse-previous-year-solved-papers-class-12-computer-science-outside-delhi-2013-28
cbse-previous-year-solved-papers-class-12-computer-science-outside-delhi-2013-29

(c) Assuming the class ANTIQUE as declared below, write a function in C++ to read the objects of ANTIQUE from binary file ANTIQUE.DAT and display those antique items, which are priced between 10000 and 15000.
cbse-previous-year-solved-papers-class-12-computer-science-outside-delhi-2013-30
cbse-previous-year-solved-papers-class-12-computer-science-outside-delhi-2013-31
cbse-previous-year-solved-papers-class-12-computer-science-outside-delhi-2013-32

Question.5. (a) Explain the concept of candidate keys with the help of an appropriate example.
Answer: A table may have more than one such attribute/ group of attribute that identifies a tuple uniquely, all such attribute(s) are known as Candidate Keys.
cbse-previous-year-solved-papers-class-12-computer-science-outside-delhi-2013-33
In the above table ID and Item both the attributes are unique in nature, so they are candidate key.
Note:
Write SQL queries for (b) to (g) and write the outputs for the SQL queries mentioned shown in (h1) to (h4) parts on the basis of tables PRODUCTS and SUPPLIERS.
cbse-previous-year-solved-papers-class-12-computer-science-outside-delhi-2013-34

(b) To display the details of all the products in ascending order of product names (i.e. P NAME).
Answer: select * from PRODUCTS order by P NAME;

(c) To display product name and price of all those products, * whose price is in the range of 10000 and 15000 (both value inclusive).
Answer: select P NAME,PRICE from PRODUCTS where PRICE>=10000 and PRICE<= 15000;

(d) To display the number of products, which are supplied supplier, i.e., the expected output should be:
501 2
502 2
503 1
Answer: select SUPCODE, count(SUPCODE) from PRODUCTS group by SUPCODE;

(e) To display the price, product name and quantity (i.e., qty) of those products which have quantity more than 100.
Answer: select PRICE, P NAME, QTY from PRODUCTS where QTY>100;

(f) To display the names of those suppliers, who are either from DELHI or from CHENNAI.
Answer: select S NAME from SUPPLIERS where
CITY=”DELHI” or CITY=”KOLKATA”;

(g) To display the name of the companies and the name of the products in descending order of company names.
Answer: select COMPANY, P NAME from PRODUCTS order by COMPANY desc;

(h) Obtain the outputs of the following SQL, queries based on the data given in tables PRODUCTS and SUPPLIERS above,
(h1) SELECT DISTINCT SUPCODE FROM PRODUCTS;
(h2) SELECT MAX(PRICE), MIN(PRICE) FROM PRODUCTS;
(h3) SELECT PRICE*QIY AMOUNT FROM PRODUCTS WHERE PID=104;
(h4) SELECT PNAME,SNAME FROM PRODUCTS P, SUPPLIERS S WHERE P.SUPCODE=S.SUPCODE AND QTY>100;
cbse-previous-year-solved-papers-class-12-computer-science-outside-delhi-2013-35

Question.6. (a) Verify the following using Boolean Laws X +-Z = X + X’.Z+Y.Z
cbse-previous-year-solved-papers-class-12-computer-science-outside-delhi-2013-36
cbse-previous-year-solved-papers-class-12-computer-science-outside-delhi-2013-37

(b) Obtain the Boolean Expression for the logic circuit shown below:
cbse-previous-year-solved-papers-class-12-computer-science-outside-delhi-2013-38

(c) Write the Sum of Product form of the function F(A, B, C) fur the following truth table representation of F.
cbse-previous-year-solved-papers-class-12-computer-science-outside-delhi-2013-39

(d) Obtain the minimal form for the following Boolean expression using Karnaugh map.
cbse-previous-year-solved-papers-class-12-computer-science-outside-delhi-2013-40
cbse-previous-year-solved-papers-class-12-computer-science-outside-delhi-2013-41

Question.7. (a) Write two advantages of using an optical fibre cable over an Ethernet cable to connect two service stations, which are 200 m away from each other.
Answer: Two using an optical fibre cable over an Ethernet cable are:
• Provides high speed.
• Electrical and Magnetic interference does not affect the transmission

(b) What is the different between HTTP and FTP?
Answer:
cbse-previous-year-solved-papers-class-12-computer-science-outside-delhi-2013-42

(c) Rovenza Communication international (RCI) is an on line corporate training provider company for IT related courses. The company is setting up their new campus in Kolltata. You as a network expert have to study the physical locations of various blocks and the number of computers to be installed. In the planning phase, provider the best possible answer for the queries (i) to (iv) raised by them.
cbse-previous-year-solved-papers-class-12-computer-science-outside-delhi-2013-43
cbse-previous-year-solved-papers-class-12-computer-science-outside-delhi-2013-44
(i) Suggest the most appropriate block, where RCI should plan to install the server.
(ii) Suggest the most appropriate block to block cable layout to connect all three blocks for efficient communication. 
(iii) Which type of network out of the following is formed by connecting the computers of these three blocks?

  • LAN
  •  MAN 
  • WAN

(iv) Which wireless channel out of the following should be opted by RCI to connect to students from all over the world?

  • Infrared
  • Microwave
  • Satellite

Answer:
(i) Faculty recording block is most appropriate block to install the server.
cbse-previous-year-solved-papers-class-12-computer-science-outside-delhi-2013-45
(iii) LAN
(iv) Satellite

(d) Write two advantages of using open source software over proprietary software.
Answer : Two advantages of using open source software aver proprietary software are:-

  • Open Source Software is software whose source code is available to customer and it can be modified and redistributed without any limitations whereas source code of proprietary Software is not available.
  • Open Source Software may come free of cost or with a payment of normal charges whereas proprietary software is neither open nor freely available.

(e) Which of the following crime(s) does not come under cybercrime?
(i) Copying some important data from a computer without taking permission from the owner of the data.
(ii) Stealing keyboard and mouse from a shop.
(iii) Getting into unknown person’s social networking account and start messaging on his behalf.
Answer: (ii) Stealing keyboard and mouse from a shop.

Idiomatic Preposition Class 8 With Answers CBSE

Looking for an easy way to Learning of new elementary english grammar and composition for class 8 answers, Solutions. You have to learn basic English Grammar topics like Tenses Verbs, Nouns, etc… In this article, we will review the best English Grammer Topics and compare them against each other

Idiomatic Preposition Class 8 With Answers CBSE PDF

Idiomatic Use Of Prepositions:

  • An idiom is an expression used in a way that is peculiar to itself either grammatically or in having a meaning that cannot be derived from the conjoined meaning of its elements.
  • It is a style or form of artistic expression.
  • In simple words, an idiom is an expression that means something more than its literal sense.
  • A preposition is a word that is used with a noun or a pronoun to show its relation to some other word in the sentence.
  • There is no hard and fast rule to explain why a particular verb or adjective takes a particular preposition after it. In English, the use of prepositions is largely a matter of convention, a matter of idiom.
  • Certain nouns, adjectives, and verbs are idiomatically used with particular prepositions and conjunctions and lend a new meaning to the sentence.

A-List of Idiomatic Prepositions

1. Abide by: a promise, a decision to stick to it: Once you have made a promise, you ought to abide by it.
2. Abound in: to be rich in: This tank abounds in fish.
3. Absorbed in: engrossed in: The singer was absorbed in music.
4. Acceptable to: Your terms are not acceptable to me.
5. Access to: approach to: A financially sound person can manage an easy access to the authorities.
6. Accordance with: According to: He helped me according to his word.
7. Account for: to explain: His extravagance accounts for his poverty.
8. Accuse of: to charge with, to blame for: He was accused of theft.
9. Act upon: to take action, etc: You must act upon this letter at once, or it will be too late.
10. Add to: to increase: Do not add injury to insult.
11. Addicted to: given to something usually evil: Being addicted to drinking and gambling, he soon squandered away his property.
12. Adhere to: stick to: We must adhere to our principles at all costs.
13. Admit to: to confess: he admitted to jumping the traffic light.
14. Admit of: to allow, leave room for: Your conduct does not admit of any excuse.
15. Afraid of: being scared of: You are afraid of your father.
16. Aim at: to point at: He aimed his rifle at the leopard.
17. Alien to: foreign to: He is alien to any feeling of self respect.
18. Alive to: conscious of: He is not alive to the weakness of his son.
19. Alive with: Connaught Place is always alive with people.
20. Angry with a person: The teacher got angry at the student.
21. Appeal to a person for or against something: He appealed to the Principal for mercy.
22. Appoint a person to a situation: He was appointed to the post of the general manager of a reputed industrial concern.
23. Approve of: Everybody approved of his plan of joining IAS.
24. Argue with a person on a subject, for or against something: Do not argue with your elders.
25. Ashamed of: One should be ashamed of ones mistake.
26. Ask about: to enquire: I asked my friend about the welfare of his family.
27. Asked for: enquire for: Go to the top floor and ask for Mr. Chopra.
28. Aspire to: He aspired to become a doctor.
29. Assure of: to confirm: I can assure of my innocence.
30. Astonished at: surprised at: His father was astonished at his cleverness.
31. Authority on: power to give order on something: Our principal is an authority on modern algebra.
32. Award to: to give a prize to: A prize was awarded to him for having won the rifle shooting championship.
33. Aware of: to have knowledge of: Are you aware of the latest changes in the syllabus.
B
34. Believe in: We believe in reasoning out things.
35. Belong to: to request: He belongs to a respectable family.
36. Beware of: cautious of: Beware of doing wrong to any man.
37. Blind of: ignorant to: Maharaja Ranjeet Singh was blind of one eye.
38. Boast of: to show off: I agree that he is talented but he boasts of his talent a bit too much.
39. Brood over: to think constantly of: It is no use brooding over past failures.
C
40. Celebrated for: famous for: Amritsar is celebrated for its Golden Temple.
41. Close to: Close to the college building is the Principal’s residence.
42. Coincide with: to accord with: His view in this matter does not coincide with me.
43. Collide with: The tonga collided with a car and the horse was killed.
44. Commence on: to begin on: Our examination commences on next Monday.
45. Commit to: to imprison: The suspect was committed to the court.
46. Consist of: to be made up of: Our family consists of six members.
47. Convenient to: You may come to see me at any time convenient to you.
48. Cope with: to contend with, to be equal to: We need three more hands to cope with the heavy rush of work.
D
49. Deal in: to trade in something: We deal in stainless steel goods.
50. Depend on: I can depend on the sincerity of my servant.
51. Devoted to: given to: He is devoted to his duty.
52. Differ from: Birds differ from beasts.
53. Disgrace to: A morally depraved officer is a disgrace to his country.
54. Disgusted at: I felt disgusted at his rude behaviour.
55. Dispose of: to get rid of, to sell: I cannot buy a new house unless I dispose of the one I am now living in.
E
56. Enquire into some matter: He enquired into the matter of murder.
57. Enter into an agreement: The two teams entered into a secret agreement.
58. Entrust with: He was entrusted with the job of making purchases from the market.
59. Envious of: Feeling envious of the brilliant result of his cousin, he also started working hard.
60. Equal to: He proved equal to the situation.
F
61. Faith in: One can work wonders in life if one has faith in oneself.
62. Faithful to: We should be faithful to our friends if we want them to be faithful to us.
63. False to: You will be false to none if you are not false to yourself.
64. Feed on: Cows feed on grass. He feeds on flattery.
65. Fight with, for: I can fight with anyone for my friend.
66. Fight against: We should not fight against our sibling.
67. Fond of: Children are fond of sweets.
G
68. Good at: Australians are very good at cricket and tennis.
69. Grieve for: a person.
70. Guilty of: You have been guilty of giving false evidence.
H
71. Hard of hearing: Having low hearing power: My friend is hard of hearing.
72. Hostile to: opposed to: When he found even his friends hostile to his proposal, he withdrew it.
I
73. Impose on, upon: A heavy responsibility was imposed on him at a very early age.
74. Incentive to: an inspiration to: Poverty often acts as an incentive to hard work.
75. Indifferent to: unconcerned with, heedless of: He is indifferent to flattery and censure.
76. Indispensable to: necessary for: You might be a very suitable candidate for the job, but do not think that you are indispensable to me.
77. Indulge in: to gratify, to give a free court to: One must not indulge in luxuries until one can afford them.
78. Inferior to: Goods made in India are no longer inferior to the imported ones.
79. Injurious to: harmful to: Indulgence in anything may prove injurious to health.
J
80. Join in: The foreign visitors joined us in the festivities.
81. Jump to the conclusion: He hastily jumped to the conclusion that I was to blame for all his losses.
K
82. Keen on: He is keen on sending his son abroad for higher studies.
83. Knock against: In the dark, he knocked against the wall.
84. Knock at: I knocked at the door but got no response.
L
85. Limit to: There is a limit to everyone’s patience.
86. Listen to: If you listen to the singing you will appreciate her voice.
M
87. Mad with: angry at: I was mad with rag at the disobedience of the students.
88. Meddle with: to interfere in: I do not like anyone to meddle with my affairs.
N
89. Necessary for: Hard work is necessary for success.
90. Negligence in: Negligence in the bringing up of children lead to sad consequences.
91. Notorious for: He is notorious for back biting.
O
92. Obedient to: We ought to be obedient to our elders.
93. Open to: Your conduct is open to questions.
94. Opposite to: There is a library opposite to our house.
P
95. Pay for: to suffer: – You will have to pay for your follies.
96. Plead with a person for something: The lawyer pleaded with the judge.
97. Possess of: He is possessed of great wealth.
98. Prey upon: Fear preyed upon my heart.
99. Prior to: before: – prior to joining this newspaper as its editor, I was working in a college.
100. Proficient in: at home in: He is proficient in Mathematics.
Q
102. Quarrel with, over: He quarreled with his friend over a trifle.
103. Qualified for: He is well qualified for the post of the manager of a restaurant.
R
104. Rely on: to depend on: If you want to rely on others, you should first be reliable to yourself.
105. Remedy against: Quinine should be used as a remedy against malaria.
106. Renowned for: famous for: He is renowned for the impartiality of his behaviour.
107. Repent of: If you want to repent of a sin, promise never to repeat it.
108. Resolve on to decide: I have resolved on setting up an export business.
109. Respectful to: Be respectful to others if you want to be respected.
110. Responsible to: to a person for something: I am responsible to God for my actions.
111. Revel in: to take keen delight in: He has been revelling in drinking and gambling.
112. Revolt against: They decided to revolt against the arbitrary decisions of the Governor.
S
113. Sacred to: These articles are sacred to the memory of my father.
114. Sentence to: The murderer was sentenced to life imprisonment.
115. Similar to: These two triangles are similar to each other.
116. Slow of: This child is slow of understanding.
117. Strike at: He struck at the snake with the stick.
118. Successful in: I am sure you will be successful in your aims.
119. Suitable to: He got a prize suitable to his age.
120. Suspicious of: distrustful of: I am suspicious of his intentions.
T
121. Taste for: liking for: – He who has no taste for music should be kept at arm’s length.
122. Tend to: Simple living and high thinking tend to a happy life.
V
123. Victim of: He was a victim of religious bigotry.
124. Victim to: A weak person falls a victim to many diseases.
125. Victory over: He won a convincing victory over his rivals.
126. Void of: deficient in: His remarks are void of sense.
W
127. Welcome to: You are welcome to our celebrations.
128. Wish for: I desperately wish for a change.
129. Worthy of: His conduct is worthy of the highest praise.
130. Wonder at: I often wonder at the spiritual stamina of Gandhi ji.
Y
131. Yearn for: to long for: Man has always yearned for eternal happiness.
132. Yield to: to surrender to: A man of spiritual strength will never yield to hardships.

Solved CBSE Sample Papers for Class 12 Political Science Set 13

Solved CBSE Sample Papers for Class 12 Political Science Set 13

[Time Allowed : 3 hrs.]                                                                                              [Maximum Marks] : 100

General Instruction:

  1. All Questions are compulsory.
  2. Question numbers 1-5 are of 1 mark each. The answers to these questions should not exceed 20 words each.
  3. Question numbers 6-10 are of 2 marks each. The answers to these questions should not exceed 40 words each.
  4. Question numbers 11-16 are of 4 marks each. The answers to these questions should not exceed 100 words each.
  5. Question numbers 17-21 are of 5 marks each. The answers to this question should not exceed 150 words.
  6. Question numbers 22-27 are of 6 marks each. The answers to this question should not exceed 150 words.

Question.1. What did the ‘Second World’ mean ?
Answer. The Second World was the group of East European countries which had been liberated by Soviet army from the fascist forces. After being liberated they came under the control of the USSR and their political and economic systems were modelled after the USSR.

Question.2. Correct the following statement and rewrite:
“Veto power of the permanent members in the Security Council is in agreement with the concept of democracy and sovereign equality.”
Answer. Veto power of the permanent members in the Security Council is a special voting power with negative nature. The Security Council can trke no decisions if any of these members uses the Veto power.

Question.3. What continues to be the most important resource in the global strategy in the sphere of economy ?
Answer. The most important resource in the global strategy in the sphere of economy is the economic flows or exchanges expressed in the form of commodities, capital, people and ideas.
It also draws our attention to the role of international institutions like the IMF and the WTO in determining economic policies across the world.

Question.4. What is meant by White Revolution ?
Answer. White Revolution signifies revolution in the field of milk production. The White Revolution was launched in Gujarat by Verghese Kurien, known as the Milkman of India.
He launched the Gujarat Cooperative Milk and Marketing Federation Ltd. to promote and channelise the milk product.
Marketing federation later on launched the well known Amul Dairy.

Question.5. Write the name of the second largest party in the Lok Sabha in the first three general elections.
Answer. The Communist Party of India was the second largest party next to Congress in the Lok Sabha in the first three general elections.

Question.6. Describe the economic system of the former Soviet Union.
Answer. The economic system of the former Soviet Union was planned and controlled by the state.

  1.  Soviet Union had a domestic consumer industry.
  2.  It had a complex communications network.
  3.  The Soviet state ensured a minimum standard of living.

Question.7. “An International organisation is not a super state with authority over its members.” Justify.
Answer. The given statement is true to the extent the international organisation is created by and responds to states. Once created, it can help member states resolve their problems peacefully.
• Actually international organisations are not the answer to every thing. But they help with matters of war and peace.
• An international organisation also helps countries cooperate to make better living conditions for us.

Question.8. “Partition of India implies administrative concern and financial strains.” Discuss.
Answer. The partition of India saw a division of properties, liabilities and assets. It also saw a political division of the country and the administrative apparatus.
• Besides there was also the division of financial assets and things like tables, chairs, typewriters, paper-clips, books and musical instruments of the police band.
• The employees of the government and the railways were also divided.
• Above all, it was a violent separation of communities who had hitherto lived together as neighbours.
All these show that the partition saw administrative concerns and financial strains.

Question.9. What does Panchsheel imply ?
Answer. The word Panchsheel signifies five principles of peaceful coexistence.
Panchsheel was signed between India and China in 1954 and is a guideline in our relations with other countries.
The “five principles of Panchsheel” are as follows :

  1. Mutual respect for each other’s territorial integrity and sovereignty
  2.  Mutual understanding
  3.  Non-intervention in each other’s internal affairs
  4. Mutual benefits and equality
  5. Peaceful co-existence

Question.10. What are the components of vital National Interests ?
Answer. The main components of National and political interest would include issues like border problems, role in the UN and NAM, territorial integrity, etc.

  1. For India, the economic interest would involve foreign exchange problems, North-south dialogue, south-south cooperation, etc.
  2.  Political dimension : National development, national stability, defence dimension i.e. national security.
  3. The defence aspect would include arms production, regional security, etc.

Question.11. In spite of having different political systems, how is democracy becoming an accepted norm in the entire region of South Asia ? Explain.
Answer. From the experience of Bangladesh and Nepal, we can say that democracy is becoming an accepted norm in the entire region of South Asia.
Pakistan and Bangladesh have experienced both civilian and military rule with Bangladesh remaining a democracy in the cold war period. Pakistan began the post-cold war period with successive democratic governments under Benazir Bhutto and Nawaz Sharif respectively. But it suffered a military coup in 1999 and has been run by a military regime since then.

  1.  Till 2006, Nepal was a constitutional monarchy .with the danger of the king taking over executive powers. In 2006, a successful popular uprising led to the restoration of democracy and reduced the king to a nominal position.
  2.  In the same way, despite many problems, Sri Lanka and India have successfully operated a democratic system. Despite many limitations India has remained a democracy throughout its existence as an independent country. The same is true of Sri Lanka.
    Hence, it is proved that the democratic system is an integral part of south Asian political model.

Question.12. Describe any four drawbacks of the Soviet system.
Answer. The Soviet system suffered from several drawbacks :

  1.  Soviet system became bureaucratic and authoritarian. Due to lack of democracy and absence of freedom, the Soviet system became bureaucratic and authoritarian. It made the life difficult for its citizens.
  2.  Soviet system became weak in economic sphere. As the Soviet economy used much of its resources in maintaining a nuclear and military arsenal and the development of its satellite states in Eastern Europe, the Soviet system became weak and its economy stagnant.
  3.  Created disparities between East and West. The Soviet system created disparities between East and West by entering into an arms race and forming military alliances.
  4.  Lack of democracy and absence of freedom of speech. In the Soviet system there was lack of democracy and absence of freedom of speech. This system was centred around the communist party and no other political party or opposition was allowed.

Question.13. Explain the concept of sustainable development.
Answer. The concept of ‘sustainable development’ signifies that development should take place without damaging the environment and development in the present should not comprdmise with the needs of the future generations.
Sustainable development is important because it results in
(i) protecting the people against pollution enduring their quality of life and health.
(ii) conserving environment which is necessary for sustainable development.
(iii) raising the standard of living of the existing population.

Question.14. How has globalisation enhanced the position of a state ?
Answer. Globalisation has enhanced the position of state in many ways.
(i) Now the state is able to rule in a better manner due to advanced technology of collecting information.
(ii) The state continues to discharge its essential functions (law and order, national security) in a very firm manner.
(iii) Even the primacy of the state continues to be an unchallenged basis of political community.
(iv) Thus, states became more powerful as an outcome of the new technology in the global Era.

Question.15. Explain India’s changing relationship with Pakistan.
Answer. Although India-Pakistan relations seem to be a story of endemic conflict and violence recently there have been a series of efforts to manage tensions and build peace.
Under the agreement of friendship signed in 2005-2006, the two countries have agreed to undertake confidence building measures to reduce the threat of war.

  1. Social activists and prominent personalities have collaborated to create an atmosphere of friendship among the people of both countries.
  2.  Leaders have met at summits to understand each other better and to find a solution to the major problems.
  3.  A number of bus routes have been opended up between the two countries.
  4.  Trade between the twq parts qf Punjab has increased substantially in the last five years.

Question.16. What is meant by ‘Common, but differential responsibilities’ in relation to environment ?
Answer. Common but differentiated responsibility signifies that states have common but differentiated responsibilities over the contribution of Global environmental degradation.
The developed countries acknowledge the responsibility that they bear in the international purrsuit of sustainable development in view of pressures their societies place on the Global environment and of the technological and financial resources they command.
Conventions and Declarations on Differentiated Responsibilities.
We can implement the ideas with the help of Conventions and Declarations like :

  1. The Rio Declaration of 1992 convened on Environmental issues says that “states shall cooperate in the spirit of Global partnership to conserve, protect and restore the health and integrity of the Earth’s ecosystem.
  2.  The 1992 United Nations Framework Convention on Climatic Change (UNFCCC) also provides that the parties should act to protect the climate system on the basis of equity and in accordance with their common but differentiated responsibilities and respective capabilities.
  3.  The Kyoto protocol is an international agreement setting targets for industrialised countries to cut their greenhouse gas emissions. The protocol was agreed to in 1997 in Kyoto in Japan, based on principles set out in UNFCCC.
    (i) The Kyoto Protocol, also enjoined on its members to protect the climatic system on the basis of equity and in accordance with their common but differentiated respective capabilities.
    (ii) Besides, certain gases the carbon dioxide, methane, hydro-fluoro carbons, etc are considered responsible for global warming, the rise in global temperature which may have catastrophic consequences for life on Earth. The Kyoto Protocol stands for the protection of Earth from all these dangerous gases.

Question.17. As a citizen of India, how would you support India’s candidature for the permanent membership of the Security Council ? Justify your proposal.
Answer. Being the citizen of India we would firmly support India’s candidature for the permanent membership of the security council on the following grounds :
• India is the second most populous country in the world comprising almost one-fifth of the world population.
• India is also the world’s largest democracy and has participated in virtually all of the initiatives of the UN.
• Its role in the UN’s peace keeping efforts is a long and substantial one.
• India’s economic emergence on the world stage is another factor that perhaps justifies our statement for India’s permanent seat in the Security Council.
• Besides, India has also made regular financial contributions to the UN and never faltered on its payments.
Apart from these, we also feel that permanent membership of the Security Council has symbolic importance and signifies a country’s growing importance in world affairs.
And this greater status is an advantage to a country in the conduct of its foreign policy. .
On the basis of above discussed grounds we can safely conclude that India has the capability to become a permanent member of the Security Council.

Question.18. Read the three extracts in the chapter from the Lok Sabha debate on the Indo-US deal. Develop any one of these into a full speech depending on a certain position on Indo- US relations.
Answer. Speech of Maj. Gen. [Retd.] B.C. Khanduri (BJP).
“Sir, I would respectfully urge this august House to pay a serious attention towards the present unipolar world led by US, the only superpower. Today the world is under the hegemony of US. But at the same time we must remember that in this unipolar world India has its own identity and has all the probability of becoming a super power.
As history tells us that although at its height hegemony seems formidable, it does not last for ever. To the contrary, balance of power politics over time, reduces the relative power of the hegemon. This is what may happen to US hegemony because of its challenges and constraints. Therefore, we feel that we should have a good and harmonious relationship with the US in. the international scenario for the mutual promotion of trade and technology. At the same time India will learn some diplomatic stands and will have a fair chance in UN. But it should not be at the cost of our security and independent identity.
With these words I conclude my speech, that by seeing the growing convergence of interests between the US and India as a historic opportunity for India, we must develop a strategy that would alow India to take advantage of US hegemony and the mutual convergences to establish the best possible options for itself.

Question.19. The peace and prosperity of countries lay in the establishment and strengthening of regional economic organisations. Justify this statement.
Answer. The above statement is very suitable to the ASEAN Security Community and the ASEAN Regional Forum which are based on the conviction that outstanding territorial disputes should not escalate into armed confrontation.
(a) The ASEAN Regional Forum [ARF] carries out coordination of security and foreign policy.
(b) At the same time ASEAN is rapidly growing into a very important regional organisation. Its Vision-2020 has defined an outward-looking role for ASEAN in the international community. This builds on the existing ASEAN policy to encourage negotiations over, conflicts in the region.
(c) In the same way, the foundation of European Union was laid for a common foreign and security policy, cooperation on justice and home affairs.
(d) The European Union has tried to expand areas of cooperation while acquiring new members especially from the erstwhile Soviet bloc.
All these examples justify the statement that the peace and prosperity of countries lies in the establishment and strengthening of regional economic organisations.

Question.20. Identify the countries where the UN peace keeping operations administered in the following years.
solved-cbse-sample-papers-for-class-12-political-science-set-13-1
Answer. (i) Haiti – 2004 (ii) Sudan – 2005
(iii) Sierra-Leone – 2006 (iv) Afghanistan – 2002

Question.21. Look at the picture given below and answer the following questions.
solved-cbse-sample-papers-for-class-12-political-science-set-13-2
(a) When did the cartoon appear in the newspapers and why ?
(b) Who was the man behind Indira Gandhi ?
(c) What exactly does the ‘Political crisis’ stand for ? Explain.
Answer. (a) This cartoon appeared few days before the declaration of emergency and captures the sense of impending political crisis.
(b) The man behind the chair of Indira Gandhi was D.K. Barooah, the.Congress President.
(c) The political crisis exactly stands for threatened low and order and the administrative process as well.
• This happened when the opposition political parties led by Jayaprakash Narayan pressed for Indira Gandhi’s rage resignations and organised a massive demonstration.
• Jayaprakash Narayan announced a nation wide Stayagraha for her resignation and asked the army, the police and government employees not to obey illegal and immoral orders.
• This too threatened the administrative activities of the government to a standstill. In retaliation the government declared a state of emergency on 25 June 1975.

Question.22. How was Non-Alignment Policy of India criticised? What is the relevance of nonalignment after the end of Cold War ?
Or
Explain as to how did the reforms initiated by Mikhail Gorbachev bring about the disintegration of the USSR.
Answer. There have been many occasions when India’s role in NAM was criticised:

  1.  India’s role in NAM was said to be ‘unprincipled’, i.e., in the name of national interest it often refused to take a firm stand on crucial international issues.
  2.  It is suggested that India was inconsistent and took contradictory postures.

For example, India signed a treaty of friendship with the USSR for 20 years. This was regarded by the outside observers as India virtually joining the Soviet Alliance system.
However, NAM contained some core values and enduring ideas and it meant that small and poor countries of the world need not become the followers of any of the super powers. Non-alignment as a strategy evolved in the Cold War context, but the end of Cold War and a Unipolar world did not see the end of the non-alignment.
Non-alignment still contains some core values and end-using ideas.

  1. Arts emphasis has shifted-from “political issues to economic issues”.
  2.  Liberalisation of third world economies for rapid development of the countries of south now remains the main concern of NAM.
  3.  Issues like democracy, disarmament, human rights and neo-colonialism are as relevant today as earlier.

With the disintegration of the erstwhile USSR, there is only one super power, therefore, it is very essential for NAM to make sincere efforts to check USA from taking unilateral decisions.- Now its emphasis has also shifted towards poverty alleviation, New International Economic Order (NIEO) based on equality, equity and justice and promotion of industrialisation.
NAM must make efforts to reshape and democratise the United Nations so that the domination of powerful countries is checked.
Besides, NAM must ensure that in the era of Globalisation, liberalism and explosion of Information Technology (IT), the developed and developing nations derive the maximum benefit and are not allowed to be exploited.
Therefore, the concept of non-alignment is applicable even in normal situation. Its essence is that every international issue is discussed on merit. This is an attitude which must be reflected in international affairs.

Or
As the question is very complicated in itself, so the answer to this question becomes more controversial in nature like:

  1.  The most basic answer seems to be that when Gorbachev carried out his reforms and loosened the system, he set in motion forces and expectations that few could have predicted. These become virtually impossible to control.
  2.  There were sections of Soviet society which felt that Gorbachev should have moved much faster and were disappointed and impatient with his methods. They did not benefit in the way they had hoped or they benefitted too slowly.
  3.  Others, especially members of the communist party, and those who were served by this system, took exactly the opposite view. They felt that their power and privileges were eroding and that Gorbachev was moving too quickly.
  4.  Hence, in this tug-of-war Gorbachev lost support on all sides and divided public opinion. Even those who were with him became disillusioned as they felt that he did not adequately defend his own policies.

Question.23. Explain India – China’s relations since Rajiv Gandhi’s visit to China in 1988.
Or
Despite the ethnic conflict, Sri Lanka has maintained a democratic political system and registered considerable economic growth and high levels of human development. Explain.
Answer. Rajiv Gandhi’s visit to China in December 1988 provided an opportunity for improvement in Indo-China relations.

  1.  After the visit both governments took mutual steps to curtail conflict and maintain peace and tranquility on the borders.
  2.  Agreements on cultural exchanges and cooperation in science and technology were signed between India and China.
  3. Four border posts were opened for trade by both the countries. This has led to an increase in the trade between the two countries, i.e., 30 per cent per year since 1999.
  4. Now in 21st century both countries have agreed to cooperate in areas of conflict between the two for instance bidding for energy at global level. Even at the Global level India and China have adopted similar policies in international economic institutions like the WTO.
  5. On the darker side, as well, the issues like China’s military relations with Bangladesh, Pakistan and Myanmar never lead to a conflict between the two.
    To conclude, we can say that increasing transportation and communication links, common economic interests and global concerns should help establish more positive and harmonious relations between China and India.

Or
Despite the ongoing ethnic conflict, Sri Lanka has registered considerable economic growth and high levels of human development through :

  1.  Bilateral trade relations
  2. Liberalising economy
  3.  Controlling the rate of growth of population
    These factors led Sri Lanka to have highest per capita gross domestic product [GDP] for many years right through the civil war. It became the first country in the region to open the economy to cope up with globalisation.
    On the other hand, despite the ravages of internal conflict, Sri Lanka has maintained a democratic political system by introducing majoritarian concept of government.

Question.24. Explain as to why the first decade of electoral competition is known as Congress system.
Or
After Independence what was the general agreement about development and what were the differences in approach to development ?
Answer. Factors for domination of political scene by the Congress :

  1. Congress is the oldest party in India. It started in 1885 and was the major party that struggled to get India independence. It produced many great leaders like Gandhi, Nehru, Rajaji, Vallabhbhai Patel, Subhas Chandra Bose. It provided strong leadership to the Indian masses.
  2.  Congress sacrificed in all possible forms to achieve independence. One can rightly say that Congress wholeheartedly fought for India’s independence.
  3.  Gandhi (the Father of the Nation) lived like an ordinary Indian and propagated ideologies of truth, non-violence, swaraj, trusteeship which the people of India thoroughly appreciated. He was the first man to launch a national movement as mass movement by bringing women, peasants and students to participate on a large scale.
  4.  Indian masses were totally impressed by the role of Congress party and having faith in the Congress they voted for it for more than four decades. The able leadership of Nehru, Indira Gandhi, Rajiv Gandhi and Narashimha Rao led India on to the path of success even though some leaders fell victim to terrorism (Indira Gandhi and her son Rajiv Gandhi were assassinated, i.e., they sacrificed their lives).
  5. The Congress after India’s independence, not only tried to solve the problems but also faced the challenges before the nation. It is not that independence meant that we had achieved everything. There were many challenges such as poverty, unemployment, low production in agriculture, industry, problem of integration of states and very importantly the refugees. It was a tough time for Congress and it managed to overcome these problems slowly but steadily.
    (vi) There was no strong political party to replace the Congress and the policies of the Congress and our relations with the neighbours favour Congress to become the dominant political party.

Or
As the concept of “development has varied scope and complex nature” so any discussion on development is bound to generate contradictions, conflicts and debates.
The first decade after independence witnessed a lot of debate around this question of development. It was common then, as it is even now, for people to refer to the “west” as the standard for measuring development.
Development was about becoming more modern and moreJike the industrialised countries of the West. It was believed that every country would go through the process of “modernisation of the West” which involved the breakdown of traditional social structures and the rise of capitalism and liberalism.
Modernisation was also associated with the ideas of growth, material progress and scientific rationality.
On the eve of independence, India had two models of modern development
solved-cbse-sample-papers-for-class-12-political-science-set-13-3
Hence, there was a debate on the selection of a model for our economy. There were some communist leaders, members of the socialist party and leaders like Nehru who supported the “Socialist model”. This reflected a broad consensus that had developed during the national movement.
All these made it clear that the task of poverty alleviation and social and economic redistribution was being seen primarily as the responsibility of the government. But at the same time there were debates among them like :

  1. For some Industrialisation seemed to be the preferred path.
  2.  For others, the development of agriculture and, in particular, alleviation of rural poverty was the priority.

Question.25. What does the term Syndicate mean in the context of the Congress Party of the sixties ? What role did the syndicate play in the Congress party ?
Or
Discuss the nature of the contest in 1971. How was the Congress (O) different from Congress (R) ?
Answer. Syndicate was the informal name given to a group of Congress leaders who were in control of the party’s organisation i.e. within the Congress.

  1.  It was led by K. Kamraj, former Chief Minister of Tamil Nadu and the then President of the Congress Party. It included powerful state leaders like S.K. Patil, S. Nijalingappa, N. Sanjeeva Reddy and Atulaya Ghosh.
  2.  In the sixties, syndicate played a decisive role in the installation of both Lai Bahadur Shastri and Indira Gandhi as the Prime Minister.
  3.  This group had a decisive say in Indira Gandhi’s first Council of Ministers and also in policy formulation and implementation.
  4.  But after the Congress split the leaders of the syndicate and those owing allegiance to them stayed with Congress (O) and a group led by Indira Gandhi formed Congress (R).
    (v) Since, it was Indira Gandhi’s Congress (R) that won the test of popularity, all these big and powerful men of Indian politics lost their power and prestige after 1971.

Or
The electoral contest of 1971 was a landmark in Indian politics. It was the restoration of Congress (R).

  1. Actually this electoral contest appeared to be loaded against Congress (R). After all the new Congress was just a faction of an already weak party. Every one believed that the real organisational strength of the Congress party was’ under the command of Congress (O).
  2.  To make matters worse for Indira Gandhi, all the major non-communist, non-Congress opposition parties formed an electoral alliance known as the “Grand Alliance”.
  3. Under the umbrella of this Grand Alliance the parties like SSP, PSP, Bharatiya Jana Sangh, Swantantra Party and the Bharatiya Kranti Dal came together to form a united front.
    Comparative Analysis between Congress (R) and Congress (O).
    The new Congress under Indira Gandhi had something that its big opponents lacked—it had an issue, an agenda, and a positive slogan.

But, the Grand Alliance did not have a coherent political programme rather it had only one common programme i.e. “Indira Hatao” (Remove Indira).

  1. In contrast to this she put forward a positive programme captured in the famous slogan: Garibi Hatao. i.e. Remove poverty.
  2.  Through the slogan of Garibi Hatao, Indira Gandhi tried to generate a support base among disadvantaged especially landless labourers, dalits, and adivasis, minorities, women and the unemployed youth.
  3.  Besides, Congress (R) focussed on the growth of the public sector, imposition of ceiling on rural land holdings and urban property, removal of disparity etc.
    Thus, the slogan of Garibi Hatao and the programmes that followed it were part of Indira Gandhi’s political strategy of building an independent nationwide political support base during the electoral contest of 1971.

Question.26. What were the main provisions of the Punjab Accord of 1985 ? How has peace been restored on secular lines in Punjab ?
Or
Explain how in the coalition era a consensus seems to have emerged among most political parties. Explain its elements also.
Answer. The Punjab Accord was an agreement signed between the then Prime Minister Rajiv Gandhi and Harchand Singh Longowal, the then President of the Akali Dal in 1985.
This agreement is known as the “Rajiv Qandhi-Longowal Accord” or the Punjab Accord. It was a step towards bringing normalcy in Punjab.
Main provisions of the Accord :

  1.  Under this accord, it was agreed that Chandigarh would be transferred to Punjab.
  2. A separate commission would be appointed to resolve the border dispute between Punjab and Haryana.
  3. A tribunal would be set up to decide the sharing of Ravi-Beas river waters among Punjab, Haryana and Rajasthan.
  4. The agreement also provided for compensation to and better treatment of those affected by militancy in Punjab and the withdrawal of the application of Armed Forces Special Powers Act in Punjab.
    However, peace did not come easily or immediately. The cycle of violence continued nearly for a decade.
    (a) Militancy and counter insurgency violence led to excesses by the police and violation of the human rights.
    (b) Politically it led to fragmentation of the Akali Dal.
    (c) Thus, the central government had to impose President’s rule in the state and the normal electoral and political process was suspended.
    (d) It was not easy to restore the political process in the atmosphere of suspicion and violence. For example when elections were held in Punjab in 1992, only 24 per cent of the electors turned out to vote.
    All this in turn added to tension between Punjab and its neighbouring states.

Or
In the midst of severe competition and many conflicts, a consensus appears to have emerged among most parties on many’crucial issues. This consensus consists of four elements.

  1.  Agreement on new economic policies : While many groups are opposed to the new economic policies, most political parties believe that these policies would lead the country to prosperity and a status of economic power in the world.
  2. Acceptance of the political and social claims of the backward castes: Political parties have recognised that the social and political claims of the backward castes need to be accepted. As a result all political parties now support reservation of seats for the backward classes in education and employment. Political parties are also willing to ensure that the OBCs get adequate share of power.
  3.  Acceptance of the role of state level parties in the governance of the country: The distinction between state level and national level parties is fast becoming less important.
  4.  Emphasis on pragmatic considerations rather than ideological positions and political alliances without ideological agreement: Coalition politics has shifted the focus of political parties from ideological differences to power sharing arrangements. Thus most parties of the NDA did not agree with the Hindutva ideology of the BJP, yet, they came together to form a government and remained in power for a full term.
    To sum up, all these are momentous changes and are going to shape politics in the near
    future.

Question.27. ‘Explain any two aspects of India’s nuclear policy.
Or
Was the Congress split in 1969 avoidable? If the split had not taken place, how could it have influenced the course of events in the 1970s?
Answer. India’s Nuclear Policy:

  1.  For a country’s development, the role of science and technology is very significant. Our
    first Prime Minister Nehru had full faith in Science and Technology for rapidly building a modern India.
  2. The Nuclear Programme was initiated in the late 1940s under the guidance of H.J. Bhabha. Former President A.P.J. Abdul Kalam had written about our Nuclear strategy in his book “India’s Vision-2020”.
  3.  When China conducted her Nuclear Test in 1964, India realised its strategic importance.
  4.  In 1968, the UN permanent members (Security Council) tried to impose Nuclear Non-Proliferation treaty on the rest of the World.
  5.  India’s first nuclear explosion was conducted in 1974. India declared that it was only for peaceful purposes.
  6.  Although there are some differences among political parties regarding external relations but they agreed on national integration, protection of national boundaries, etc. This could be clearly seen when we faced three wars during the decade (1962-1972).
  7. India refused to sign the CTBT (Comprehensive Test Ban Treaty) in 1995.
  8.  India also conducted a series of nuclear tests in May 1998, demonstrating its capacity to use nuclear energy for military purposes.

Or
The political events which followed the elections of 1967 indicated that the split in the Congress was not avoidable.
The Congress retained power at the centre but with a reduced majority.
The Congress lost power in many states and non-Congress governments were formed as coalitions.
It should be noted that the results proved that the” Congress could be defeated at the elections. The political events could be analysed under the following headings

  1. Indira vs. the ‘Syndicate’.
  2.  Presidential Election, 1969.
  1.  Indira vs. the Syndicate : Indira Gandhi faced a serious problem not from outside but from her own party. ‘Syndicate’ was a group of Congress leaders who were in control of the party’s organisation. Some of the prominent leaders were K. Kamaraj, S.K. Patil, S. Nijalingappa, etc. They wanted to make Indira Gandhi the Prime Minister and expected her to follow their advice. Indira Gandhi wanted to be independent of the ‘Syndicate’ and to regain the power for Congress which was lost in the 1967 elections. She launched a 10-Point Programme and a government policy with left orientation.
  2.  Presidential Election 1969: The factional rivalry between the Syndicate and Indira Gandhi came in the open in 1969 when the President Zakir Hussain died.
    The Syndicate leaders proposed Neelam Sanjeeva Reddy as their official candidate without even consulting Mrs Indira Gandhi. She supported the then Vice President V.V. Giri to file his nomination as an independent candidate.
    The syndicate leader S. Nijalingappa issued a ‘whip’ asking all the Congress MPs and MLAs to vote in favour of Sanjeeva Reddy but ultimately V.V. Giri emerged victorious. She also announced many popular policy measures like nationalisation of 14 banks and the abolition of the privy purses (special privileges) given to the former princes. She raised the slogan ‘Garibi Hatao’ and returned to power with a solid majority in 1971 elections. In case the split did not take place the progressive measures might not be achieved. The split was not avoidable and the events after 1969 proved to be beneficial to the people of India.
Class 8 Geography Chapter 6 Extra Questions and Answers Human Resource

Class 8 Geography Chapter 6 Extra Questions and Answers Human Resource

Get ready to embark on an enlightening expedition through the annals of human resource, where the echoes of human potential resonate with the spirit of inclusive development. Together, we’ll explore the profound insights of “Class 8 Geography Chapter 6 Extra Questions and Answers Human Resource” and gain a deeper appreciation for the transformative power of human capital in shaping our world. Read this also Extra Questions for Class 8 Social Science .

Class 8 Geography Chapter 6 Extra Questions and Answers Human Resource

Human Resource Class 8 Extra Questions and Answer Geography Chapter 6 Very Short Answers Type

Question 1.
What does the population pyramid tells about dependents of a country?
Answer:
Population pyramid tells us how many dependents are there in a country. There are two dependents – young and elderly dependents.

Question 2.
What could be an interesting way of studying the population composition of a country?
Answer:
An interesting way of studying the population composition of a country is by looking at the population pyramid, also called an age-sex pyramid.

Question 3.
How crowded a country is, has little to do with its level of economic development. Give example to show its validity?
Answer:
For example, both Bangladesh and Japan are very densely populated but Japan is far more economically developed than Bangladesh. This is the perfect example for the above’s validity.

Question 4.
Define immigration.
Answer:
When a person enters a new country it is termed as immigration.

Question 5.
What does Emigrations mean?
Answer:
Emigration means when a person leaves a country.

Question 6.
What is the general trend of international migration?
Answer:
The general trend of international migration is from the less developed nations to the more developed nations in search of better employment opportunities.

Question 7.
Who are the Emigrants?
Answer:
The emigrants are the people who leave a country.

Question 8.
What is the meaning of population Density?
Answer:
Population density is the number of people living in a unit area of the earth’s surface.

Question 9.
Which is the ultimate resource?
Answer:
Human Resource.

Question 10.
Which of the two continents are there in which almost 3 quarters of world’s people live?
Answer:
Asia and Africa.

Question 11.
What does industrial area provide?
Answer:
It provides employment opportunities.

Question 12.
What is the most preferrable topography for humans?
Answer:
Plains

Question 13.
Which area of the world has sparse population when it comes to water availability?
Answer:
Deserts.

Question 14.
What led the people to settle in South Africa.
Answer:
Diamond mines of South Africa.

Question 15.
In which year the world’s population reached 3 billion?
Answer:
In 1959.

Question 16.
What was the world’s population in 1804?
Answer:
The world’s population reached one billion.

Question 17.
Why is the population growth slow in the countries like United Kingdom?
Answer:
Because of both low death and low birth rates.

Human Resource Class 8 Extra Questions and Answer Geography Chapter 6 Short Answers Type

Question 1.
What is meant by the term population density and what is the average density of population in whole world and in India?
Answer:
Population density is the number of people living in a unit area of the earth’s surface. It is normally expressed as per sq km. The average density of population in the whole world is 51 persons per sq km. and the average density of population in India is 382 persons sq km. South Central Asia has the highest density of population.

Question 2.
How can we say that people’s are the nation’s greatest resources and human resource is the ultimate resource?
Answer:
People are the nation’s greatest resources as nature’s bounty becomes significant only when people find it useful. It is people with their demands and abilities that turn, them into resources. Hence, human resource is the ultimate resource. Healthy, educated and motivated people develop resources as per their requirements.

Question 3.
What do you know about Human Resource Development (HRD) ministry under the government of India?
Answer:
The Government of India has a ministry of Human Resource Development. The Ministry was created in 1985 with an aim to develop people’s skills. This just shows how important people are as a resource for the country.

Question 4.
What do you mean by the pattern of population distribution? How is world population distributed?
Answer:
The way in which people are spread across the earth’s surface is known as the pattern of population distribution. More than 90% of the world’s population lives in about 30 per cent of the land’s surface. The distribution of world population is extremely uneven.

Question 5.
Define the following:

  1. Birthrate
  2. Death rate
  3. Migrations
  4. Life expectancy

Answer:

  1. Birth Rate: Births are usually measured using the birth rate, i.e. the number of live births per 1,000 people.
  2. Death Rate: Deaths are usually measured using the death rate, i.e. the number of deaths per 1,000 people.
  3. Migrations: It is the movement of people in and out of an area.
  4. Life expectancy: It is the number of years that an average person can expect to live.

Human Resource Class 8 Extra Questions and Answer Geography Chapter 6 Long Answers Type

Question 1.
Define population change. How do the following factors affect the distribution of population?
(i) Topography
(ii) Climate
(iii) Soil
(iv) Water
(v) Minerals.
Answer:
The population change refers to change in the number of people during a specific time. The world population has not been stable.
This is how the following factors affect the distribution of population.

  1. Topography: People always prefer to live on plains rather than mountains and plateaus because these areas are suitable for farming, manufacturing and service activities.
  2. Climate: People usually avoid extreme climates that are very hot or very cold.
  3. Soil: Fertile soils provide suitable land for agriculture. Fertile plains are densely populated around different parts of the world.
  4. Water: People prefer to live in the areas where freshwater is easily available. The river valleys of the world are densely populated while deserts have spare population.
  5. Minerals: Areas with mineral deposits are more populated.

Picture Based Questions Class 8 Geography Chapter 6 Human Resource

Look at the picture given below and answer the following questions:

Question 1.
Class 8 Geography Chapter 6 Extra Questions and Answers Human Resource img-1
(i) When does the population increase?
(ii) When does the condition of steady population arise?
(iii) When does the population decrease?
Answer:
(i) When birth rate is more than death rate.
(ii) When birth rate and death rate both stay the same.
(iii) When death rate is more than birth date.

Map Skills Class 8 Geography Chapter 6 Human Resource

Question 1.
Mark the Annual rates if natural increase in the following colours.
(i) High (2 – 2.9%) – with red colour.
(ii) 0 or decreases – with green colour.
Answer:
Class 8 Geography Chapter 6 Extra Questions and Answers Human Resource img-2

CBSE Sample Papers for Class 12 Political Science All India – 2013

CBSE Sample Papers for Class 12 Political Science All India – 2013

Time Allowed: 3 Hours                                                                                          Maximum Marks: 100
General Instructions:

  1. All questions are compulsory.
  2. Question Numbers 1 to 10 are of one mark each. The answers to these questions should not exceed 20 words each.
  3.  Question Numbers 11 to 20 are of two marks each. The answers to these questions should not exceed 40 words each.
  4. Question Numbers 21 to 30 are of four marks each. The answers to these questions should not exceed 100 words each.
  5. Question Numbers 31 to 35 are of six marks each. The answers to these questions should not exceed 150 words each.
  6. Question Number 35 is based on the map. Write the answer in your Answer-Book.

SET -I

Question.1. What is the main reason for beginning of the US hegemony in 1991 ? 1
Question.2. Define the ‘Bandwagon strategy’. 1
Question.3. Correct the following sentence and rewrite in your answer book:
The International Monetary Fund has five member countries and they enjoy equal status. 1
Question.4. Mention any two new threats to the security of a state. 1
Question.5. Who was popularly known as ‘the milkman of India’ ? 1
Question.6. What was ‘Bombay Plan’ ? 1
Question.7. Name the two main corttenders who contested the election for the post of President of India in 1969.1
Question.8. Who led the Railway strike in 1974 ? What was its main demand ? 1
Question.9. Which organisation of Assam led the movement against foreign nationals in 1979 ? 1
Question.10. Shetkari Sanghatana and Rayata Sangha belonged to which two respective states ?1
Question.11. Which were the two important features of the Soviet System ? 2×1 = 2
Question.12. State any two consequences of the ‘largest garage sale’ in history. 2×1 = 2
Question.13. What is meant by ‘hegemony’ ? 2
Question.14. Name any four main organs of the United Nations. 2
Question.15. What is meant by the non-traditional notion of security ? 2
Question.16. Mention any two merits of Green Revolution. 2×1 = 2
Question.17. Mention any two important features of Bhartiya Jana Sangh’s ideology. 2×1 = 2
Question.18. What were the two main consequences of Indo-Pakistan conflict of 1971 ? 2×1 = 2
Question.19. Highlight any two issues that dominate the politics of North-East India. 2×1 = 2
Question.20. What does a coalition government mean ? Mention any one example of such a government. 1 + 1 = 2
Question.21. Why is the policy of non-alignment of India criticised ? Explain any two reasons. 2×2 = 4
Question.22. Explain any four factors which make the European Union a highly influential organisation. 4×1 = 4
Question.23. Explain the hegemony of United States as a Hard Power. 4
Question.24. Describe India’s relations with China from Independence to 1962. 4
Question.25. State any four criteria that have been proposed in the recent years for new permanent and non-permanent members of the UN Security Council. 4×1 = 4
Question.26. Explain any four factors responsible for the development of globalisation. 4×1 = 4
Question.27. Who founded the Swatantra Party in 1959 ? Describe any three policies and programmes of this party. 1 + 3 = 4
Question.28. Explain India’s Nuclear policy. 4
Question.29. What reasons, do you think, were responsible for the declaration of emergency in 1975 ? Examine any two reasons. 2×2 = 4
Question.30. In the given political outline map of India, four States have been marked (A), (B), (C),and (D). Identify them with the help of the information given below and write their correct names in your answer-book with their respective serial number and the alphabet concerned : 4×1 = 4
(i) The State from where Chipko Movement began.
(ii) The State related to ‘Operation Blue Star’
(iii) The State which was liberated from the control of Portugal in December 1961.
(iv) The State which has been given a special status under Article 370.
cbse-sample-papers-for-class-12-political-science-all-india-2013-1
Question.31. Study the passage given below carefully and answer the questions that follow:
The collapse of Communism was followed in most of these countries by a painful process of transition from an authoritarian socialist system to a democratic capitalist system … Privatisation of state assets and corporate ownership patterns were to be immediately brought in.
Questions: 3×2 = 6
(i) Why has the process of transition been described as painful ?
(ii) Which political system existed before the transition and which system replaced it, if any ?
(iii) What does privatisation imply ?
Or
Study the passage given below and answer the questions that follow :
The two superpowers were keen On expanding their spheres of influence in different parts of the world. In a world sharply divided between the two alliance systems, a state was supposed to remain tied to its protective superpower to limit the influence of the other superpower and its allies…. Most countries of Western Europe sided with the US and those of the Eastern Europe joined the Soviet camp.
Questions:
(i) Name the two superpowers.
(ii) Why did the allies want to remain tied with one of the superpowers ?
(iii) Why did the superpowers want to bring other countries into their camps ?
Question.32. Explain any three environmental concerns in global politics. 3×2 = 6
Or
Explain the concept of globalisation and any two reasons for resistance to it. 2 + 4 = 6
Question.33. “Congress had remained a social and ideological coalition for a long period.” Justify the statement. 6
Or
Assess the outcomes of the early phase of planned development in India.
Question.34. Mention any six steps taken for the restoration of dominance of the Congress Party after the 1971 elections. 6×1 = 6
Or
Mention any six factors responsible for the defeat of the Congress Party in 1977 elections.
Question.35. What are popular movements ? Describe any four benefits of popular movements. 2+4 =6
Or
Highlight any three issues on which a broad agreement has emerged among most of the political parties in India. 3×2 = 6

SET – II

Question.1. What was the objective of founding the European Union ? 1
Question.5. Name any two opposition parties which were part of an electoral alliance in 1971, known as ‘Grand Alliance’. 1
Question.7. Name the present General Secretary of the United Nations who belongs to Asia. 1
Question.9. What was significant about PC Mahalanobis ? 1
Question.10. When was the first democratic election to Sikkim Assembly held and which party swept the elections ? 2
Question.13. Mention the names of any four permanent member states of the UN Security Council. Commissioner ? 1 + 1 = 2
Question.22. Describe any four main events of India’s war with Pakistan in 1965. 4 x 1=4
Question.24. Explain the new economic policies of China since 1978. 4
Question.26. Explain any two causes which led to the formation of ASEAN. 2×2 = 4
Question.28. Explain any four steps taken by the Government of India to curb the emission of green house gases. 4x 1 = 4
Question.35. Describe the growth and characteristics of the Farmers’ Movement led by Bharatiya Kisan Union since 1988. 6
Or
Describe the Assam Movement against the outsiders from 1979 to 1985.

SET – III

Question.2. Name the leader who had given up active politics, but was invited to lead the students’ movement in Bihar in 1974. 1
Question.3. Mention the main function of the World Trade Organisation. 1
Question.9. Who is the ex-officio chairperson of the Planning Commission of India ? 1
Question.13. Which are the two forms of cooperative security as per its traditional notion ? 2×1 = 2
Question.17. Name the pro-Soviet and pro-China factions of the Communist Party of India after the split. 2×1 = 2
Question.22. Explain any four causes of India’s war with Pakistan in 1971. 4×1 = 4
Question.24. How is democratisation going on in Nepal since 1990 ? 4
Question.26. What does SAARC stand for ? Highlight its any three main objectives. 1 + 3 = 4
Question.28. What is the meaning of ‘global commons’ ? Explain any two reasons for the dwindling of global commons. 2×2 = 4
Question.35. What is Sardar Sarovar Project ? What are the achievements of this movement ? Why is it being opposed ? 3×2 = 6
Or
Describe any two secessionist movements of North-East India. 2×3 = 6

CBSE Previous Year Solved Papers Class 12 Chemistry Outside Delhi 2011

CBSE Previous Year Solved  Papers  Class 12 Chemistry Outside Delhi 2011

Time allowed: 3 hours                                                                                           Maximum Marks: 70

General Instructions:

  1.  All questions are compulsory. 
  2.  Questions number 1 to 5 are very short-answer questions and carry 1 mark each.
  3.  Questions number 6 to 10 are short-answer questions and carry 2 marks each.
  4.  Questions number 11 to 22 are also short-answer questions and carry 3 marks each.
  5.  Questions number 23 is a value based question and carry 4 marks.
  6.  Questions number 24 to 26 are long-answer questions and carry 5 marks each. 
  7.  Use log tables, if necessary. Use of calculators is not allowed.

SET I

Question.1.Define ‘order of a reaction’.
Answer : The sum of powers of the concentrations of the reactants in the rate law expression is called the order of a reaction.

Question.2.What is meant by ‘shape selective catalysis’?
Answer : Catalysis using selective absorbents like zeolites as catalyst is called shape selective catalysis. In this catalysis small sized molecules are absorbed in the pores and cavities of zeolites.

Question.3.Differentiate between a mineral and an ore.
Answer:
cbse-previous-year-solved-papers-class-12-chemistry-outside-delhi-2011-1

Question.4. What is meant by ‘lanthanoid contraction’?
Answer : The lanthanoid contraction refers to the steady and regular decrease in atomic radii along the period from La+3 to Lu+3.

Question.5. Write the IUPAC name of the following compound:
CH2 = CHCH2 Br
Answer: 3-Bromo-l-propene

Question.6. Draw the structure of 4-chloropentan-2-one.
Answer.
cbse-previous-year-solved-papers-class-12-chemistry-outside-delhi-2011-2

Question.7. How would you convert ethanol to ethane?
Answer:
cbse-previous-year-solved-papers-class-12-chemistry-outside-delhi-2011-3

Question.8. Rearrange the following in an increasing order of their basic strengths:
cbse-previous-year-solved-papers-class-12-chemistry-outside-delhi-2011-4

Question.9. Explain how you can determine the atomic mass of an unknown metal if you know its mass density and the dimensions of unit cell of its crystal.
cbse-previous-year-solved-papers-class-12-chemistry-outside-delhi-2011-5
cbse-previous-year-solved-papers-class-12-chemistry-outside-delhi-2011-6

Question.10.Calculate the packing efficiency of ametal crystal for a simple cubic lattice.
cbse-previous-year-solved-papers-class-12-chemistry-outside-delhi-2011-7
Question.11. State the following:

  1.  Raoult’s law in its general form in reference to solutions.
  2. Henry’s law about partial pressure of a gas in a mixture.

Answer :

  1.  Raoult’s law : The partial vapour pressure of each component of a solution is equal to the vapour pressure of pure component multiplied by its mole fraction in the solution
    cbse-previous-year-solved-papers-class-12-chemistry-outside-delhi-2011-8
  2.  Henry’s law : It states that the partial pressure of gas in vapour phase (P) is direedy proportional to mole fraction of gas (x) in the solution and is expressed as
    cbse-previous-year-solved-papers-class-12-chemistry-outside-delhi-2011-9

Question.12. What do you understand by the rate law and rate constant of a reaction? Identify the order of a reaction if the units of its rate constant are:

  1. L-1mol s-1
  2. L mol s-1

Answer : Rate law of a chemical reaction is the expression relating the rate of reaction to the concentrations or pressures of various reactants taking part in the reaction.
The rate of reaction at unit concentration of all reactants is known as the rate constant (K)

  1.  Zero order
  2. Second order

Question.13. The thermal decomposition of HCO2 H is a first order reaction with a rate constant of 2.4 x 10-3 s-1 at a certain temperature. Calculate how long will it take for three- fourths of initial quantity of HCO2H to decompose, (log 0.25 = -0.6021)
cbse-previous-year-solved-papers-class-12-chemistry-outside-delhi-2011-10

Question.14. Describe the principle controlling each of the following
processes:

  1.  Vapour phase refining of titanium metal
  2. Troth floatation method of concentration of a sulphide ore

Answer :

  1. Titanum is converted to its volatile form which is evaporated and then decomposed to give pure titanium.
  2.  The ore particles get adsorbed on oil droplets and come to the surface where they can be collected as froth gangue is wetted by water and gets settledown. ,

Question.15. How would you account for the following:

  1.  Cr2+ is reducing in nature while with the same d-orbital configuration (d4) Mn3+ is an oxidizing agent.
  2. In a transition series of metals, the metal which exhibits the greatest number of oxidation states occurs in the middle of the series.

Answer :

  1.  Cr2+has configuration d4 which easily changes to d3 due to stable half filled ftg orbitals. Therefore, Cr2+ is reducing agent and Mn3+ easily changes to Mn2+ and acts as an oxidizing agent.
  2. Due to presence of more unpaired electrons and more number of partially filled orbitals in the middle of a transition series,these metals exhibit the greatest number of oxidation states.

Question.16. Complete the following chemical equations :
cbse-previous-year-solved-papers-class-12-chemistry-outside-delhi-2011-11
State reasons for the following :
(i) Cu (I) ion is not stable in an aqueous solution.
(ii)Unlike Cr3+, Mn2+, Fe3+ and the subsequent other M2+ ions of the 3d series of elements, the 4d and the 5d series metals generally do not form stable cationic species.
Answer:
cbse-previous-year-solved-papers-class-12-chemistry-outside-delhi-2011-12
(ii) Because of Lanthanoid contraction, expected increase in size does not occur. That is why they do not form stable cations.

Question.17. Explain what is meant by the following:

  1.  Peptide linkage
  2. Pyranose structure of glucose

Answer :

  1. Peptide linkage is the peptide bond formed between amino acids. It is a covalent bond formed between amino group of one molecule and carboxylic acid group of another molecule, causing the release of one molecule of water.
    cbse-previous-year-solved-papers-class-12-chemistry-outside-delhi-2011-13
  2.  Cyclic structure of glucose is called pyranose because it resembles pyran ring. Its structure includes a six membered ring with 5-carbon atoms and one oxygen atom having no double bonds.
    cbse-previous-year-solved-papers-class-12-chemistry-outside-delhi-2011-14

Question.18. Write the main structural difference between DNA and RNA. Of the four bases, name those which are common to both DNA and RNA.
Answer:
cbse-previous-year-solved-papers-class-12-chemistry-outside-delhi-2011-15

Question.19. A solution prepared by dissolving 8.95 mg of a gene fragment in 35.0 mL of water has an osmotic pressure of 0.335 torr at 25°C. Assuming that the gene fragment is a non-electrolyte, calculate its molar mass.
cbse-previous-year-solved-papers-class-12-chemistry-outside-delhi-2011-16

Question.20. Classify colloids where the dispersion medium is water. State their characteristics and write an example of each of these classes.
OR
Explain what is observed when

  1. an electric current is passed through a soln.
  2. a beam of light is passed through a soln.
  3. an electrolyte (say NaCl) is added to ferric hydroxide sol

Answer : Colloids can be classified into two types where the dispersion medium is water. They are as follows :

  1.  Hydrophillic colloids or Lyophillic: The substances when mixed with dispersion medium form colloidal solution direcdy are called hydrophilic colloids. They are quite stable, reversible
    solutions and can’t get precipated easily, e.g., Gum starch, etc.
  2. Hydrophobic colloids or Lyophobic : The substances which do not form colloidal solution with dispersion medium are called hydrophobic colloids. They are unstable, irreversible solutions and can be easily precipitated, e.g., Metals and their sulphides.

OR

  1. When electric current is passed through a solution, then positively charged ions move towards cathode and negatively charged ions move towards anpde. Then they get coagulate. This is known as Electrophoresis.
  2. When a beam of strong light is passed through solution, light gets scattered by the colloidal particles and path of light becomes visible. This is known as Tyndall effect.
  3. When NaCl is added to ferric hydroxide sol then a negatively charged solution is obtained with absorption of OH ion.

Question.21. How would you account for the following:

  1.  H2S is more acidic than H2O.
  2. The N-O bond in NO2 is shorter than the N-O bond in NO3 
  3. Both O2 and F2 stabilize high oxidation states but the ability of oxygen to stabilize the higher oxidation state exceeds that of fluorine.

Answer:

  1. Size of sulphur is large than oxygen due to which S-H bond length increase and hence the bond dissociation energy of S-H is less than O-H. Therefore S-H easily loses H+ and is more acidic than H2O.
  2. Due to tendency of nitrogen to form pπ-pπ  multiple
    bonds, there is a difference in N-O bond lengths of NO2 and NO2 .
  3. Due to the property of oxygen to form double bonds with the metal atoms, oxygen stabilizes the higher oxidation state even more than fluorine.

Question.22. Explain the following terms giving a suitable example in each case:

  1.  Ambident ligand
  2. Denticity of a ligand
  3. Crystal field splitting in an octahedral field

Answer :

  1. Ligands which can ligate to the central atom in two places ligands such as SCN which can. attach at either S atom or N atom are called ambidentate ligand.
  2. The number of donor atoms of a ligand when bound with central atom in a coordination complex is called its denticity.
  3. The splitting of five degenerate d-orbitals in two sets, one with three orbitals and another with two orbitals is known as crystal field splitting.

Question.23. Rearrange the compounds of each of the following sets in order of reactivity towards SN2 displacement:

  1. 2-Bromo-2-methylbutane, 1-Bromopentane, 2-Bro- mopentane
  2. 1 -Bromo-3-methylbutane, 2-Bromo-2-methyl-butane, 2-Bromo-3-methylbutane
  3. 1-Bromobutane, l-Bromo-2, 2-dimethylpropane, 1- Bromo-2-methylbutane, l-Bromo-2, 2-dimethylpropane.

Answer:

  1. 1-bromopentane > 2-bromopentane > 2-bromo- 2-methylbutane.
  2. 1-bromo-3 methylbutane > 2-bromo-3-methylbutane > 2-bromo-2-methyl butane.
  3. 1-bromobutane>1 -bromo-3-methylbutane> 1-bromo-2-, 2-dimethylpropane >1-bromo-2, 2-dimethyl propane.

Question.24. How would you obtain the following:
(i) Benzoquione from phenol
(ii)2-Methylpropan-2-ol from methylmagnesium bromide
(iii)Propan-2-ol from propene
Answer:
cbse-previous-year-solved-papers-class-12-chemistry-outside-delhi-2011-17
cbse-previous-year-solved-papers-class-12-chemistry-outside-delhi-2011-18
cbse-previous-year-solved-papers-class-12-chemistry-outside-delhi-2011-19

Question.25. State reasons for the following:

  1. pKbvalue for aniline is more than that for methylamine.
  2. Ethyiamine is soluble in water whereas aniline is not soluble in water.
  3.  Primary amines have higher boiling points than tertiary amines.

Answer :

  1. Higher value of  pKb means lower basicity, therefore, aniline is less basic than methylamine because in aniline the lone pair of electrons on N atom gets delocalized over the benzene ring and remain unavailable for protonation due to resonance but this is absent in methylamine.
  2. Ethyiamine forms H-bonds with water therefore, it is soluble in water but aniline does not form H-bonds with water due to its larger hydrocarbon part and is insoluble in water.
  3. In primary amines, two H-atoms are attached to N-atom and they undergo intermolecular H-bonding, but tertiary amines due to the absence of H-atom on N-atom do not undergo H-bonding . Therefore, primary amines have higher boiling point than tertiary amines.

Question.26. Draw the structures of the monomers of the following polymers:

  1. Polythene
  2. PVC
  3. Teflon

Answer :

  1. ethene CH2 = CH2.
  2. Vinyl chloride CH2 = CHCl
  3. Tetraflouroethylene CF2 = CF2

Question.27. What are the following substances? Give one example of each.

  1.  Food preservatives
  2. Synthetic detergents
  3. Antacids

Answer :

  1.  Chemicals added to food to prevent its spoilage by killing or preventing the growth of microorganisms like bacteria, yeasts and moulds. e.g. sodium benzoate.
  2. Synthetic detergents are sodium or potassium salts of long chain sulphonic acids. They don’t precipitate in hard water. e.g. Sodium laurylsulphate.
  3. Antacids are chemicals consumed to get relief from acidity in the stomach by neutralizing excess acid, e.g, milk of magnesia.

Question.28. (a) What type of a battery is lead storage battery? Write the anode and cathode reactions and the overall cell reaction occurring in the operation of a lead storage battery.
(b) Calculate the potential for half-cell containing 0.10 M K2Cr2O7 (aq), 0.20 M Cr3+ (aq) and 1.0 x 10-4 M H+ (aq) The half-cell reaction is and the standard electrode potential is given as E° = 1.33 V
cbse-previous-year-solved-papers-class-12-chemistry-outside-delhi-2011-20
(a) How many moles of mercury will be produced by electrolyzing 1.0 M Hg(N03)2 solution with a current of 2.00 A for 3 hours? [Hg(NO3)2 = 200.6 g mol1]
(b) A voltaic cell is set up at 25°C with the following half-cells Al3+ (0.001 M) and Ni2+ (0.50 M). Write an equation for the reaction that occurs when the cell generates an electric current and determine the cell potential.
cbse-previous-year-solved-papers-class-12-chemistry-outside-delhi-2011-21
Answer :
(a) Lead storage battery is a secondary cell (rechargable). The electrode reaction is as follows :
cbse-previous-year-solved-papers-class-12-chemistry-outside-delhi-2011-22
cbse-previous-year-solved-papers-class-12-chemistry-outside-delhi-2011-23
cbse-previous-year-solved-papers-class-12-chemistry-outside-delhi-2011-24

Question.29. (a) Draw the structures of the following molecules:
cbse-previous-year-solved-papers-class-12-chemistry-outside-delhi-2011-25
OR
(a) What happens when
(i) Chlorine gas is passed through a hot concentrated solution of NaOH?
(ii) Sulphur dioxide gas is passed through an aqueous solution of a Fe(III) salt?
(b) Answer the following :
(i) What is the basicity of H3PO3and why?
(ii)Why does fluorine not play and role of a central atom in interhalogen compounds?
(iii)Why do noble gases have very low boiling points?
(b)(i) Two, because H3PO3 has two replaceable H+ ions as one hydrogen is attached directly to phosphorus and can’t be released as a proton.
(ii)Since fluorine is the most electro-negative halogen, it does not acts as a central atom in interhalogen compounds.
(iii)Noble gases are monoatomic with weak Vander Waals forces of attraction and hence have low boiling points.
cbse-previous-year-solved-papers-class-12-chemistry-outside-delhi-2011-26
cbse-previous-year-solved-papers-class-12-chemistry-outside-delhi-2011-27

Question.30. (a) Illustrate the following name reactions :
(i) Cannizzaro’s reaction
(ii)Clemmensen reduction
(b) How would you obtain the following :
(i) But-2-enal from ethanal
(ii)Butanoic acid from butanol
(iii)Benzoic acid from ethylbenzene
OR
(a) Give chemical tests to distinguish between the following:
(i) Benzoic acid and ethyl benzoate
(ii)Benzaldehyde and acetophenone
(b) Complete each synthesis by giving missing reagents or products in the following :
cbse-previous-year-solved-papers-class-12-chemistry-outside-delhi-2011-28
Answer : (a)
(i) Aldehydes having no a-hydrogen atom undergoes self oxidation and reduction on treatment with concentrated alkali and produce alocohol and carboxylic acid salt.
cbse-previous-year-solved-papers-class-12-chemistry-outside-delhi-2011-29
cbse-previous-year-solved-papers-class-12-chemistry-outside-delhi-2011-30
cbse-previous-year-solved-papers-class-12-chemistry-outside-delhi-2011-31

SET II

Note : Except for the following questions, all the remaining question have been asked in previous set.

Question.2.What are lyophobic colloids? Given one example for them.
Answer : Colloids in which the colloidal particles have no affinity for the dispersion medium and they do not form colloidal solution are lyophobic colloids, like, Al(OH)3 and  AS2S3 sols.

Question.3. Why is it that only sulphide ores are concentrated by ‘froth floatation process’?
Answer : Due to the affinity of heavy oil droplets to adsorb sulphide particles. The ore particles come on the surface as froth, gangue particles are wetted by water and get settle down.

Question.5. Write the IUPAC name of the following compound:
cbse-previous-year-solved-papers-class-12-chemistry-outside-delhi-2011-32
Answer: 3-Bromo-2-methyl prop-l-ene

Question.6. Draw the structure of 2, 6-Dimethylphenol.
Answer:
cbse-previous-year-solved-papers-class-12-chemistry-outside-delhi-2011-33

Question.9. Define the following terms in relation to crystalline solids:

  1.  Unit Cell
  2. Coordination number

Answer :

  1.  Unit Cell: The smallest three dimensional unit of a lattice which when repeated in three dimensions generates the entire lattice is called the unit cell, e.g., Cubic unit cell.
  2.  Co-ordination number : Coordination number of an atom in a crystal is the number of immediate neighbours of the atom in that crystal, e.g., Coordination number of hexagonal (hep) structure isT2.

Question.12. A reaction is of second order with respect to a reactant. How is the rate of reaction affected if the concentration of the reactant is reduced to half ? What is the unit of rate constant for such a reaction ?
cbse-previous-year-solved-papers-class-12-chemistry-outside-delhi-2011-34

Question.14. Describe the principle controlling each of the following processes:

  1.  Zone refining of metals
  2. Electrolytic refining of metals

Answer :

  1.  Zone refining of metals : It is based on the principle that impurities are more soluble in molten state of metal than in the solid state,
  2.  Electrolytic refining : Impure metal is made of anode, thin sheet of pure metal is made of cathode and a salt of the metal is used as an electrolyte. On passing current, metal from anode goes into the solution and ions in the solution reduce on cathode leading to deposition of pure metal.

Question.15. Explain giving a suitable reason for each of the following :

  1.  Transition metals and their compounds are generally found to be good catalysts.
  2. Metal-metal bonding is more frequent for the 4d and the 5d series of transition metals than that for the 3d series.

Answer :

  1.  Due to presence of vaccant orbitals and tendency to form large number of oxidation states, transition metals have a high tendency to form complexes and hence acts as a catalysts.
  2.  In transition metals of 4d and 5series the 4d and 5d electrons are at greater distance from the nucleus therefore they are less tighdy held to . the atom by the nucleus and hence contribute more to metallic bonding , as compared to transition metals of 3d series.

Question.19. What mass of NaCl must be dissolved in 65.0 g of water to lower the freezing point of water by 7.50 °C? The freezing point depression constant (Kf) for water is 1.86°C/m. Assume Van’t Hoff factor for NaCl is 1.87. (Molar mass of NaCl = 58.5 g)
cbse-previous-year-solved-papers-class-12-chemistry-outside-delhi-2011-35

Question.22. Write the structures and names of all the stereoisomers of the following compounds :
cbse-previous-year-solved-papers-class-12-chemistry-outside-delhi-2011-36
cbse-previous-year-solved-papers-class-12-chemistry-outside-delhi-2011-37
cbse-previous-year-solved-papers-class-12-chemistry-outside-delhi-2011-38
cbse-previous-year-solved-papers-class-12-chemistry-outside-delhi-2011-39

Question.27. (a) Differentiate between a disinfectant and an antiseptic. Give one example of each.
(b) What is tincture of iodine and what is it used fori
Answer: (a)
cbse-previous-year-solved-papers-class-12-chemistry-outside-delhi-2011-40
Answer : (b) 2-3% solution of iodine in alcohol and water is called tincture iodine and are widely is used as an antiseptic

SET III

Note : Except for the following questions, all the remaining question have been asked in previous sets.

Question.1. Define ‘activation energy’ of a reaction
Answer: Activation energy is the minimum energy that need to be provided to the reactants for the reaction to take place.

Question.2. What is meant by ‘reverse osmosis’?
Answer : When a pressure larger than osmotic pressure is applied the solvent will flow from the solution into the pure solvent through the semipermeable membrane, called reverse osmosis.

Question.3. What type of ores can be concentrated by magnetic separation method?
Answer : Ores with different magnetic properties and impurities magnetic in nature can be concentrated by magnetic separation method.
e.g. Chromite (Fe O. Cr2O3), Magnetite (Fe3O4)

Question.14. Describe the principle controlling each of the following processes:

  1. Preparation of cast iron from pig iron.
  2. Preparation of pure alumina (Al2O3) from bauxite ore.

Answer :

  1.  Preparation of cast iron from pig iron : The iron obtained from blast furnace is called pig iron. Cast iron is prepared by meltiilg pig iron with scrap iron and coke using hot air blast. It has slightly lower carbon content (about 3%). It is extremely hard and brittle.
  2.  Preparation of pure alumina from bauxite ore : The principal ore of aluminium is bauxite Al2O3.xH20. Bauxite is concentrated by digesting the powdered ore with concentrated solution of NaOH at 473-523K. The Al2O3 is leached out as sodium aluminate. The sodium aluminate is neutralized by passing CO2gas and hydrated Al2O3is precipitated which is filtered, dried and heated to give pure Al2O3.
    cbse-previous-year-solved-papers-class-12-chemistry-outside-delhi-2011-41

Question.15. Explain giving reasons :

  1.  Transition metals and their compounds generally  exhibit a paramagnetic behaviour.
  2. The chemistry of actinoids is not so smooth as that of lanthanoids.

Answer :

  1. Transition metals and their compounds exhibit a paramagnetic behaviour due to presence of unpaired electrons in the Penultimate shell of d-orbital.
  2.  Lanthanoids show limited number of oxidation states, +2, +3, +4 because of large energy gap between 4f and 5d subshells. Actinoids show a number of oxidation states +4,+5, +6, +7 due to small energy difference between 5f, 6d and 7s subshells.

Question.18. Write such reactions and facts about glucose which cannot be explained by its open chain structure.
Answer : Limitations of open chain structure of glucose :

  1.  Glucose does not form NaHS03 as addition product.
  2. Glucose penta-acetate does not react with NH2OH due to absence of aldehyde group.

Question.21. How would you account for the following:

  1.  NF3 is an exothermic compound but NCl3is not.
  2. The acidic strength of compounds increase in the order:
  3.  SF6 is kinetically inert.

Answer :

  1. In NF3, fluorine molecule, F-F bond enthalpy is very high, so large amount of energy is released when NF3 is formed but this is not in the case of NCl3.
  2. As electronegativity increases in the same period from left to right so their electronegativity is in the increasing order P < S < Cl. Therefore, acidic strength increases in the order,
    PH3 < H2S < HCl
  3. SF6 is protected by six F atoms and hence does not allow to attack on sulphur atom.

Question.22 Write the state of hybridization, the shape and the magnetic behaviour of the following complex entitles :
cbse-previous-year-solved-papers-class-12-chemistry-outside-delhi-2011-42
cbse-previous-year-solved-papers-class-12-chemistry-outside-delhi-2011-43
Question.26. Write the names and structures of the monomers of the following polymers:
(i)Buna-S
(ii)Dacron
(iii)Neoprene
cbse-previous-year-solved-papers-class-12-chemistry-outside-delhi-2011-44

CBSE Sample Papers for Class 12 Political Science Delhi – 2013

CBSE Sample Papers for Class 12 Political Science Delhi – 2013

Time Allowed: 3 Hours                                                                                          Maximum Marks: 100
General Instructions:

  1. All questions are compulsory.
  2. Question Numbers 1 to 10 are of one mark each. The answers to these questions should not exceed 20 words each.
  3.  Question Numbers 11 to 20 are of two marks each. The answers to these questions should not exceed 40 words each.
  4. Question Numbers 21 to 30 are of four marks each. The answers to these questions should not exceed 100 words each.
  5. Question Numbers 31 to 35 are of six marks each. The answers to these questions should not exceed 150 words each.
  6. Question Number 35 is based on the map. Write the answer in your Answer-Book.

SET -I

Question.1.What is meant by ‘9/11’ in the context of USA ? 1
Question.2.Correct the following statement and rewrite :
ASEAN stands for ‘Association of South East African Nations’. 1
Question.3.How many Permanent Members and how many Non-permanent Members does the UN Security Council have ? 1
Question.4.What is meant by ‘Security’ ? 1
Question.5.What is the ‘Two Nation Theory’ advanced by the Muslim League ? 1
Question.6.Name the founder President of the Congress Socialist Party. What name was given to this Party after 1948 ? 1
Question.7.After which General Election in India did Congress Party lose its dominance for the first time at the Centre ? 1
Question.8.After the death of Lai Bahadur Shastri, which two leaders of the Congress Party contested against one another to become leader of the Congress Parliamentary Party ? 1
Question.9.Name the popular movement which demanded that no forest exploiting contract should be given to any outsider. 1
Question.10.From which year did the era of Coalition Governments at the Centre begin in India ? 1
Question.11.Name the two superpowers responsible for Cold War. When did the world become unipolar? 1+1=2
Question.12.What is meant by ‘Shock Therapy’ ? 2
Question.13.When was the ASEAN Regional Forum established ? What was its main objective ? 2
Question.14.Explain the traditional concept of ‘Security’. 2
Question.15.Why are International Organizations like the UN required ? 2
Question.16.Which four Princely States of India initially resisted to join-the Indian Union ? 2
Question.17.Which are the two models of development ? Which model of development was adopted by India ? 1+1=2
Question.18.What is India’s Policy of Non-alignment ? 2
Question.19.Mention any two issues of concern related to development projects such as Sardar Sarovar Project. 2×1=2
Question.20. Highlight any two recommendations of the Mandal Commission. 2×1=2
Question.21. Explain any two reasons for the disintegration of the USSR. 2×2=4
Question.22. List any four ‘Directive Principles of State Policy’ given in the Constitution of India for the promotion of international peace and security. 4×1=4
Question.23. Describe any two constraints on American Hegemony. 2×2=4
Question.24. Describe any two major issues of conflict between India and Pakistan leading to the War of 1971. 2×2=4
Question.25. Identify and explain any four new sources of threat to security. 4×1=4
Question.26. Explain any four consequences of globalization. 4×1=4
Question.27. Explain any four reasons for the dominance of the Congress Party in the first three General Elections. 4×1=4
Question.28. Where and when was the organization ‘Dalit Panthers’ formed ? Describe any three of its activities. 1+3=4
Question.29.Why is the ‘Emergency’ and the period around it known as the period of Constitutional crisis ? Explain. 4
Question.30.In the given Political Outline Map of Europe, four member countries of the European Union have been marked A, B, C and D. Identify them with the help of information given below and write their correct names in your answer-book along with their respective serial number and the alphabet concerned 4
(i) An older member of the European Union located between Portugal and France.
(ii) An older member of the European Union located near Belgium and Netherlands.
(iii) Two new members of the European Union.
cbse-sample-papers-for-class-12-political-science-delhi-2013-1
Question.31. Study the passage given below carefully and answer the questions that follow:
The best way to respond to regional aspirations is through democratic negotiations rather than through suppression. Look at the situation in the eighties-militancy had erupted in Punjab; problems were persisting in the North-East; students in Assam were agitating; Kashmir Valley was on the boil. Instead of treating these as simple law and order problems, the Government of India reached negotiated settlement with regional movements.
Questions:
(i) How are regional aspirations dangerous for the unity of the country ?
(ii) What is meant by democratic negotiations ?
(iii) Who was leading the agitation in Assam ?
(iv) What steps were taken by the Government of India to respond to the regional aspirations? 6
Or
Study the passage given below and answer the questions that follow:
In fact, the BSP under Kanshi Ram’s leadership was envisaged as an organization based on pragmatic politics. It derived confidence from the fact that the Bahujans (SCs, STs, OBCs and religious minorities) constituted the majority of the population and were a formidable political force on the strength of their numbers.
Questions:
(i) Which organization was formed by Kanshi Ram ?
(ii) Give the full form of BSP.
(iii) Name any two religious minorities.
(iv) Why are the Bahujans considered a formidable political force ?
Question.32. ‘States have common but differentiated responsibilities towards environment.’ Analyse the statement giving suitable examples. 6
Or
Analyse India’s stand on environmental issues.
Question.33. Explain any three consequences of partition of British India in 1947. 3×2=6
Or
Explain the process and basis of the reorganisation of States of Indian Union. 6
Question.34. ‘Was the declaration of emergency in 1975 necessary ?’ Support your answer with any three suitable arguments. 3×2=6
Or
Explain any three lessons learnt from the emergency imposed in 1975.
Question.35. What was the Soviet System ? Assess any four features of the Soviet System. 2+4=6
Or
Examine the relevance of the Non-aligned Movement after the Cold War era.

SET-II

Question.2. Name the political party of India which was founded by Acharya Narendra Dev before independence. 1
Question.6. What is meant by ‘Congress Syndicate’ ? 1
Question.8. Mention any one objective of the United Nations. 1
Question.10. The States of which region of India are referred to as the ‘Seven Sisters’ ? 1
Question.13. What is meant by ‘Operation Enduring Freedom’ ? 2
Question.17. Highlight the importance of popular movements. 2
Question.22. Explain any four problems before the Election Commission of India for holding the First General Election. 4×1=4
Question.24. Describe the Nuclear Policy of India. 4
Question.26. Describe any four new econorinic policies of China to make it grow at a faster rate. 4×1=4 Question.28. How has technology contributed to globalization ? Explain. 4
Question.34. Examine any six consequences of disintegration of the Soviet Union. 6×1=6
Or
Assess any three reforms proposed by the United Nations Conference on Trade and Development (UNCTAD) as new trade policy for development. 3×2=6

SET -III

Question.2. When was the Constitution of India ready and signed by members of the Constituent Assembly ? When did come into effect ? V2+Vi=l
Question.6. What is meant by Privy Purse ? 1
Question.8. Mention any one function of the World Bank. 1
Question.10. Mention any one point of agreement included in the Rajiv Gandhi-Longowal Accord. 1
Question.13. Before 1971, which two reasons were a source of resentment among the people of East Pakistan (now Bangladesh) against West Pakistan ? 2×1=2
Question.17. Mention any two demands of the Bharatiya Kisan Union. 2×1=2
Question.22. Explain any two differences between the First Five Year Plan and the Second Five Year Plan. 2+2=4
Question.24. Describe the contribution of Jawaharlal Nehru to shape properly the Foreign Policy of India. 4
Question.25. Identify any four new sources of threat to security and explain them. 4×1=4
Question.26. Highlight any four features of’ASEAN’. 4×1=4
Question.28. Explain .any four reasons due t<? which globalization is resisted. 4×1=4
Question.32. Why did the superpowers have military alliances with smaller countries ? Explain any three reasons. 3×2=6
Or
Give any three examples to show that most of the former Soviet Republics were prone to conflicts and tensions.

CBSE Previous Year Solved Papers Class 12 Maths Outside Delhi 2015

CBSE Previous Year Solved  Papers Class 12 Maths Outside Delhi 2015

Time allowed: 3 hours                                                                                          Maximum Marks : 100
General Instructions:

  1. All questions are compulsory.
  2. Please check that this question paper contains 26 questions.
  3. Questions 1-6 in Section A are very short-answer type questions carrying 1 mark each.
  4. Questions 7-19 in Section B are long-answer I type questions carrying 4 marks each.
  5. Questions 20-26 in Section C are long-answer II type questions carrying 6 marks each.
  6. Please write down the serial number of the question before attempting it.

SET I

SECTION – A

Question.1. Write the value of ∆ =
cbse-previous-year-solved-papers-class-12-maths-outside-delhi-2015-1
Solution.
cbse-previous-year-solved-papers-class-12-maths-outside-delhi-2015-2

Question.2. Write the sum of the order and degree of the following differential equation:
cbse-previous-year-solved-papers-class-12-maths-outside-delhi-2015-3
Solution.
cbse-previous-year-solved-papers-class-12-maths-outside-delhi-2015-4

Question.3. Write the integrating factor of the following differential equation:
cbse-previous-year-solved-papers-class-12-maths-outside-delhi-2015-5
Solution.
cbse-previous-year-solved-papers-class-12-maths-outside-delhi-2015-6

Question.4.
cbse-previous-year-solved-papers-class-12-maths-outside-delhi-2015-7
Solution.
cbse-previous-year-solved-papers-class-12-maths-outside-delhi-2015-8

Question.5. Write a unit vector perpendicular to both the vectors
cbse-previous-year-solved-papers-class-12-maths-outside-delhi-2015-9
Solution.
cbse-previous-year-solved-papers-class-12-maths-outside-delhi-2015-10

Question.6. The equations of a line are 5x – 3 = 15y + 7 = 3- 10z. Write the direction cosines of the line.
Solution.
cbse-previous-year-solved-papers-class-12-maths-outside-delhi-2015-11

SECTION – B

Question.7. To promote the making of toilets for women, an organization tried to generate awareness through (i) house calls (ii) letters, and (iii) announcements. The cost for each mode per attempt is given below: (i)Rs 50 (ii) Rs 20 (iii)Rs 40
The number of attempts made in three villages X, Y and Z are given below:
(i)    (ii)      (iii)
400  300    100
300  250      75
500  400       150
Find the total cost incurred by the organization for the three villages separately, using matrices.
Write one value generated by the organization in the society.
Solution. The number of attempts made in three villages X, Y and Z can be represented by the 3 x 3 matrix.
cbse-previous-year-solved-papers-class-12-maths-outside-delhi-2015-12
Hence the total cost incurred by the organization for the three villages separately are Rs 30,000, Rs 23,000 and Rs 39,000.
The organization in the society generated the value of cleanliness for the women welfare.

Question.8.
cbse-previous-year-solved-papers-class-12-maths-outside-delhi-2015-13
Solution. Given
cbse-previous-year-solved-papers-class-12-maths-outside-delhi-2015-14
OR
cbse-previous-year-solved-papers-class-12-maths-outside-delhi-2015-15
Solution.
cbse-previous-year-solved-papers-class-12-maths-outside-delhi-2015-16

Question.9. Using properties of determinants, prove the following:
cbse-previous-year-solved-papers-class-12-maths-outside-delhi-2015-17
Solution.
cbse-previous-year-solved-papers-class-12-maths-outside-delhi-2015-18
cbse-previous-year-solved-papers-class-12-maths-outside-delhi-2015-19

Question.10. Find the adjoint of the matrix.
cbse-previous-year-solved-papers-class-12-maths-outside-delhi-2015-20
Solution.
cbse-previous-year-solved-papers-class-12-maths-outside-delhi-2015-21
cbse-previous-year-solved-papers-class-12-maths-outside-delhi-2015-22
cbse-previous-year-solved-papers-class-12-maths-outside-delhi-2015-23

Question.11. Show that the function f(x) = | x-1 | + | x +1|, for all x ϵ R, is not differentiable at the points x=-1 and x = 1.
Solution.
cbse-previous-year-solved-papers-class-12-maths-outside-delhi-2015-24
cbse-previous-year-solved-papers-class-12-maths-outside-delhi-2015-25
cbse-previous-year-solved-papers-class-12-maths-outside-delhi-2015-26

Question.12.
cbse-previous-year-solved-papers-class-12-maths-outside-delhi-2015-27
Solution.
cbse-previous-year-solved-papers-class-12-maths-outside-delhi-2015-28
cbse-previous-year-solved-papers-class-12-maths-outside-delhi-2015-29

Question.13.
cbse-previous-year-solved-papers-class-12-maths-outside-delhi-2015-30
Solution.
cbse-previous-year-solved-papers-class-12-maths-outside-delhi-2015-31

Question.14.
cbse-previous-year-solved-papers-class-12-maths-outside-delhi-2015-32
Solution.
cbse-previous-year-solved-papers-class-12-maths-outside-delhi-2015-33
cbse-previous-year-solved-papers-class-12-maths-outside-delhi-2015-34
cbse-previous-year-solved-papers-class-12-maths-outside-delhi-2015-35
OR
cbse-previous-year-solved-papers-class-12-maths-outside-delhi-2015-36
Solution.
cbse-previous-year-solved-papers-class-12-maths-outside-delhi-2015-37
cbse-previous-year-solved-papers-class-12-maths-outside-delhi-2015-38

Question.15.
cbse-previous-year-solved-papers-class-12-maths-outside-delhi-2015-39
Solution.
cbse-previous-year-solved-papers-class-12-maths-outside-delhi-2015-40
cbse-previous-year-solved-papers-class-12-maths-outside-delhi-2015-41

Question.16.
cbse-previous-year-solved-papers-class-12-maths-outside-delhi-2015-42
Solution.
cbse-previous-year-solved-papers-class-12-maths-outside-delhi-2015-78
cbse-previous-year-solved-papers-class-12-maths-outside-delhi-2015-43

Question.17.
cbse-previous-year-solved-papers-class-12-maths-outside-delhi-2015-44
Solution.
cbse-previous-year-solved-papers-class-12-maths-outside-delhi-2015-45

Question.18. Find the equation of a line passing through the point (1,2, -4) and perpendicular to two lines
cbse-previous-year-solved-papers-class-12-maths-outside-delhi-2015-46
Solution. Let the direction ratios of required line be a, b, c, t since, the line is perpendicular to
cbse-previous-year-solved-papers-class-12-maths-outside-delhi-2015-47
OR
Find the equation of the plane passing through the points (-1, 2, 0), (2, 2, -1) and parallel to the line
cbse-previous-year-solved-papers-class-12-maths-outside-delhi-2015-48
Solution.
cbse-previous-year-solved-papers-class-12-maths-outside-delhi-2015-49
cbse-previous-year-solved-papers-class-12-maths-outside-delhi-2015-50

Question.19. Three cards are drawn successively with replacement from a well shuffled pack of 52 cards. Find the probability distribution of the number of spades. Hence find the mean of the distribution.
Solution. Let X denote the number of spades when three cards are drawn, then, X is a random variable that can take values 0,1,2,3:
Let E be the event when spade card is drawn,
cbse-previous-year-solved-papers-class-12-maths-outside-delhi-2015-51
cbse-previous-year-solved-papers-class-12-maths-outside-delhi-2015-52
OR
For 6 trials of an experiment, let X be a binomial variate which satisfies the relation 9P(X = 4) = P(X =2). Find the probability of success.
Solution. Let p denote the probability of getting success and q be the probability of failure.
cbse-previous-year-solved-papers-class-12-maths-outside-delhi-2015-53

SECTION – C

Question.20. Consider f: R+ -> [- 9, ∞ ] given by f(x) = 5x + 6x – 9. Prove that f is invertible with
cbse-previous-year-solved-papers-class-12-maths-outside-delhi-2015-54
Solution.
cbse-previous-year-solved-papers-class-12-maths-outside-delhi-2015-55
cbse-previous-year-solved-papers-class-12-maths-outside-delhi-2015-56
OR
cbse-previous-year-solved-papers-class-12-maths-outside-delhi-2015-57
Solution.
cbse-previous-year-solved-papers-class-12-maths-outside-delhi-2015-58

Question.21. Find the value of p for which the curves x2= 9p(9 – y) and x2 = p(y +1) cut each other at right angles.
cbse-previous-year-solved-papers-class-12-maths-outside-delhi-2015-59
cbse-previous-year-solved-papers-class-12-maths-outside-delhi-2015-60

Question.22. Using integration, prove that the curves y2 = 4x and x2 = 4y divide the area of the square bounded by x = 0, x = 4, y = 0 and y = 4 into three equal parts.
Solution.
cbse-previous-year-solved-papers-class-12-maths-outside-delhi-2015-61
cbse-previous-year-solved-papers-class-12-maths-outside-delhi-2015-62
cbse-previous-year-solved-papers-class-12-maths-outside-delhi-2015-63

Question.23.
cbse-previous-year-solved-papers-class-12-maths-outside-delhi-2015-64
Solution.
cbse-previous-year-solved-papers-class-12-maths-outside-delhi-2015-65
cbse-previous-year-solved-papers-class-12-maths-outside-delhi-2015-66
OR
Find the particular solution of the differential equation (tan-1y -x) dy = ( 1 + y2) dx, given that x = 1, when y = 0.
Solution.
cbse-previous-year-solved-papers-class-12-maths-outside-delhi-2015-67
cbse-previous-year-solved-papers-class-12-maths-outside-delhi-2015-68
cbse-previous-year-solved-papers-class-12-maths-outside-delhi-2015-69
cbse-previous-year-solved-papers-class-12-maths-outside-delhi-2015-70
cbse-previous-year-solved-papers-class-12-maths-outside-delhi-2015-71

Question.24. Find the distance of the point P(3,4, 4) from the point, where the line joining the points A(3, – 4, – 5) and B(2, -3,1) intersects the plane 2x + y + z = 7.
Solution. Equation of the line joining the points A(3, – 4, – 5) and B(2, – 3,1) is
cbse-previous-year-solved-papers-class-12-maths-outside-delhi-2015-72

Question.25. A company manufactures three kinds of calculators A, B and C in its two factories I and II, The company has got an order for manufacturing at least 6400 calculators of Kind A, 4000 of kind B and 4800 of kind C. The daily output of factory I is of 50 calculators of kind A, 50 calculators of kind B, and 30 calculators of kind C. The daily output of factory II is of 40 calculators of kind A, 20 of kind B and 40 of kind C. The cost per day to run factory I is Rs 12,000 and of factory II is Rs 15,000. How many days do the two factories have to be in operation to produce the order with the minimum cost ? Formulate this problem as an LPP and solve it graphically.
Solution.
cbse-previous-year-solved-papers-class-12-maths-outside-delhi-2015-73
cbse-previous-year-solved-papers-class-12-maths-outside-delhi-2015-74
cbse-previous-year-solved-papers-class-12-maths-outside-delhi-2015-75

Question.26. In a factory which manufactures bolts, machines A, B and C manufacture respectively 30% 50% and 20% of the bolts. Of their outputs 3,4 and 1 percent respectively are defective bolts. A bolt is drawn at random from the product and is found to be defective. Find the probability that this is not manufactured by machine B.
Solution.
cbse-previous-year-solved-papers-class-12-maths-outside-delhi-2015-76
cbse-previous-year-solved-papers-class-12-maths-outside-delhi-2015-77